ORAL PATHOLOGY

अब Quizwiz के साथ अपने होमवर्क और परीक्षाओं को एस करें!

At the age of nine, poet Lucy Grealy was diagnosed with Ewing's sarcoms of the jaw. Although rare in females, and rare in the jaw, Ewing's sarcoma most often presents radiographically as: Multiple radiolucent/radiopaque lesions resembling "cotton ball" or "cotton wool" appearance . Multiple "punched-out" radiolucencies . "Moth-eaten" expansion destructive radiolucencies of medulla and erosion of the cortex with expansion Lytic lesion that rnay be ill-defined or sharply defined

"Moth-eaten" expansion destructive radiolucencies of medulla and erosion of the cortex with expansion Eling's sarcoma is an uncommon, highly lethal, round cell sarcoma ofbone ofuncer- tain origin. The most common sites for Ewing's sarcoma are the pelvis, the thigh, and the trunk ofthe body. When the jaws are involved, there is predilection for the ramus ofthe mandible. Ninety percent of Ewing's sarcoma occur between the ages of5 and 30 years. and over 607o affect males. See picture # 39 in booklet Pain. usually ofan intermittent nature, and swelling ofthe involved bone are ollen the ear- liest clinical signs and symptoms of Ewing's sarcoma. Involvement of the mandible ol maxilla may result in facial deformity, destruction of alveolar bone with loosening of reerh. and mucosal ulcers. Feveq leukocytosis, raised ESR, and anemia are also present. Radiographically. the most characteristic appearance is that ofa moth-eaten d€structive radiolucency of medulla and erosion ofthe cortex with expansion. A variable periosteal "onion-skin" reaction may also be seen. See picture #40 in booklet Histologically, it is often difficult to distinguish this tumor from a neuroblastoma or a reticulum cell sarcoma, however, the cells of Ewing's sarcoma contain glycog€n. The highly malignant nature ofthis sarcoma is reflected in its propensity for metastasis, especially to lungs, other bones, and the lymph nodes. Multiple method treatment proto- cols. including surgery or radiation for local control and chemotherapy for systemic mi- crometastases have dramatically improved the formerly dismal l07o 5-year swvival rate to a 60olo 5-year suryival rate. Important: The most common osseous malignancies are osteosarcomas, followed by chondrosarcomas, fibrosarcomas and Ewing's sarcoma.

At which stage is metastasis most likely for malignant melanoma "Horizontal" growth phase "Circular" growth phase "Vertical" growth phase "Radial" growth phase

"Vertical" growth phase Melanomas develop initially as a flat phase without competence for metastasis called the "radial growth phase". This refers to the initial growth of a melanona in a horizon- tal plane. It is clinically macular or only slightly elevated. They then may evolve focally an elevated part, the vertical growth phase, with metastatic competence. The "vertical" growth phase is the phase that begins when neoplastic cells populate the underlying dermis. lt is characterized clinically by an increase in size, a change in color, nodularity and, at times, ulceration. Metastasis is possible once the melanoma reaches this phase. Overall, patients with vertical growth phase melanomas have a cure rate of 7070. Note: This phase predoninates in nodular melanoma. Radial growth phase melanomas, although invasive, have a cure rate that approaches 1009'o with surgery alone. Radial growth phase melanomas include the superficial spread- ing. lentigo maligna, and acral lentiginous melanomas. Generally, the radial growth phase is described by the A,B,C,D's of melanoma: A : asynmetry B : border irregularity C : color variability (brolvtr, black, blue, gray, pink) D = diameter of (olexl greater than 1/4 inch Remember: Malignant melanoma is an uncommon neoplasm of the oral mucosa. It ex- hibits a definite predilection for the palate and the maxillary gingiva/alveolar ridge. Untortunately. oral mucosa melanomas have a dismal prognosis. The five-year survival rate for such tumors is < 20olo (/br skin lesions it i,s > 65%o).

Whats the estimated toxic dose of fluoride ingestion? l-2 mg/kg 5- l0 mg/kg 8-10 mg/kg 12-15 mg/kg

5- l0 mg/kg *** Death may result from ingesting as little as 2 g offluoride in an adult and l6 mg,&g in chil- dren. Symptoms may appear with 3-5 mg/kg offluoride. S) mptoms of acute fluoride poisoning include nausea, abdominal pain, vomiting, diarrhea, conr ulsions. and hypotension. The treatment for acute fluoride poisoning includes: 1) Call poison control center, 2) Monitor vital iigns. 3) Initiate basic life support as needed and 4) Get patient to the hospital Fluoride poisoning may be ac\te (caused b1' a single larye close offluoride) or chronic (caused hr long-term ingestion offluoride). The characteristic signs ofchronic fluoride poisoning are: l. Osteosclerosis of the bones: which results fiom long-term ingestion of water with l0 to 15 ppm offluoride. :. Dental fluorosis (enamel hypoplasia): which is due to fluoride intake during the calcification stage oftooth development. This can occur in permanent and deciduous teeth. \ote: [t has been estimated that the average American diet contains about 0.2 to 0.3 mg of fluoride per day. lf I ppm of fluodde is added to the drinking watel about I to 2 mg of flu- oride will be added to the diet daily. Balance studies have shown that when quantities offlu- oride ingested do not exceed 4 to 5 mg daily, little is retained by the body. The finding in- dicates the safety of the preventive dentistry programs based on the addition of fluoride to drinking water in concentrations of approximately 1 ppm. Important: Fluoride normally accumulates slowly in bones as a person ages. However, if ingested in very high amounts, it accumulates rapidly. The intake of calcium in high doses rvill reduce the absomtion ofdietarv fluoride.

A 55-year-old patient comes into your office for routine dental work. You see that he has a tooth fracture (due to decay) of tooth #31, A smooth, firm, asymptomatic lesion is noted on the lateral border of the tongue adjacent to the sharp enarnel of tooth #31. The patient states that the lesion has been there for years and is annoying because sometimes he will bite it accidentally. Name this most frequently encountered intraoral benign neoplasm of connective tissue origin, . A leiomyosarcoma A traumatic fibroma A leiomyorna A rhabdomyoma

A traumatic fibroma Traumatic fibroma, also know as initation fibroma, focal fibrous hyperplasia, and h)?erplastic scar, is a reactive lesion caused usually by chronic trauma to oral mucorls mcmbranes. Ovcrcxubcrant fibrous connective tissue repair results in a clinically evident submucosal mass.

A newborn baby girl was delivered via caesarian section due airway patency concerns. During ultrasound, there was the discovery of a tumor of the oral cavity. Upon delivery, the pink, compressible tumor of the anterior maxilla was deemed to be a congenital epulis of the newborn. This lesion is composed of cells that are identical to those of the: . A traumatlc neuroma . A schwannoma . A granular cell myoblastoma . A lipoma

A granular cell myoblastoma (aka granular cell tumor) The congenital epufis of the newborn (also called congenital gingival granular cell tumor).lt rsually appears on the gingiva (usually anteriof ofnewborns. It presents as a noninflamed, pedunculated or broad-based mass. The maxillary gingiva is more often involved than the mandibular gingiva, and ferrales are affected more than males. The treatment is surgical excision with little possibility ofrecurence. See pictur€ #24 in booklet The granular cell myoblastoma is a rare neoplasm of unknown etiology. Most re- searchers believe its origin is from the Schrvrum cell. It presents as an uninflamed. asymp- tomatic mass less than 2 cm in diameter. The most common location in the head and neck region is the tongue. It may affect any age group and females seem to be affected more than men. See picture #25 in booklet Important: Both of these lesions are identical histologically. They both contain granu- lar cells, however, the congenital epulis of the newborn does not exhibit overlaying pseudoepitheliomatous hyperplasia. The pseudoepitheliomatous hyperplasia of the or erlying epithelium is frequently seen in the granular cell myoblastoma.

The abnormal loss of tooth structure due to non masticatory physical friction is referred to asa Erosion Abfraction Attrition Abrasion

Abrasion Types ofabrasion: . l. Toothbrush abrasion: most olten results in V-shaped wedges at the cervical margin in the canine and premolar areas. lt is caused by the use ofa hard toothbrush and/or a horizontal brushing stroke and/or a gritty dentifrice. 2. Occlusal abrasion: results in flattened cusps on all posterior teeth and wom incisal edges. lt results from the chewing or biting ofhard foods or objects and chewing tob- acco. Attrition is the wearing away of enamel and dentin due to the normal function or most commonly, due to the excessive grinding or gdtting together oftecth by the patient lreJbrced to a.r bruxism). The most noticeable effects ofattrition are polished facets, flat incisal edges. discolored surfaces ofthe teeth and exposed dentin. Facets usually develop on the linguoin- cisal of the maxillary central incisors, the facioincisal of the mandibular canines and the linguoincisal ofthe maxillary canines. See picture #14 in booklet . Erosion is the loss of tooth structure from non-mechanical means. It can result from drinking acidic liquids or eating acidic foods. It is common in bulimic individuals as a re- sult of regurgitated stomach acids. It affects smooth and occlusal surlaces. See picture #12 in booklet . Abfraction lesions are cervical erosive lesions that can not be attributed to any partic- off starting at the base ofthe tooth and exposing the gum Iine ofthe tooth to excessive wear See picture #11 in booklet ular cause: causing the enamel to "pop''

All of the following statements are true EXCEPT one. which one is the EXCEPTION Acute leukemias have a slow onset and progression Acute leukemias are characterized by the appearance of immature, abnormal cells in the bone marrow and peripheral blood and frequently in the liver, spleen, lymph nodes, and other parenchymatous organs The clinical picture of acute leukemias are marked by the effects ol anemia, which is usually severe (/ittigue, molaise), an absence of functioning granulocles (proneness to inlbction and inflammation), and thrombocytopenia (hemorrlrugic diathesis) The spleen and liver usually are moderately enlarged, while enlarged lymph nodes are seen mainly in acute lymphocytic leukemia. Fever and a very high ESR are found Leukocyte counts vary greatly in the acute leukemias

Acute leukemias have a slow onset and progression This is false; acute leukemias have a rapid ons€t and progression. -{cute leukemia is characterized by malignant proliferation olwhite blood cell precursors I hle.\ts) in bone marrow and lymph tissue and their accumulation in peripheral blood, bone rnanow, and body tissues. Leukemic cells inhjbit normal bone marrow production rrf en throcytes. platelets, and immune function. Orher important features ofacute leukemia: . -A.brupt onset (fe11,noltlsl with sudden high fever, weakness, malaise, severe anemia, and . . -ueneralized lymphadenopathy; bone and joint pain common in children. Principal organ involved: bone marrow (along witlt the spleen and liver) Petechiae and ecchymoses in skin and mucous membranes, hemorrhage from vari- ous sites: bacterial infections common. . r Laboratory findings: leukocytosis 30,000-1000,000 per cu.mm. with immature forms nn eloblasts and 1.,-mphoblasts) pred,ominating. . ln 7-i9i ofthe cases olacute lymphocytic leukemia, the lymphocytes are neither B nor T-cells and are called . "null cells. " L ntreated patients die within six months; with intensive therapy rutliation, and bone marrow transplanls) remissions lasting up to five years may be obtained; death is usually due to a hemorrhage (broin) or a superimposed bacterial infection.

A 17-year-old patient of yours comes in for a routine examination. A head and neck examination reveals multiple cysts of the skin. Her panoramic exams have always shown multiple impacted teeth and today shows multiple radiopacities of the jaws especially at the angle of the mandible. You suspect Gardners syndrome. What complication should she most be concerned with when consulting her physician? Odontomas Osteomas Adenocarcinoma Epidermoid cysts GI polyposis

Adenocarcinoma Gardner's syndrome is an autosomal dominant disorder and is characterized by intestinal polyposis, multiple osteomas, fibromas ofthe skin, epidermal and trichilemrnal cysts, im- pacted pemanent and supernumerary teeth, and odontomas. The most serious complica- tion of Gardner's syndrome is the multiple polyps that affect the large intestine. The inevitable outcome ofthis disease is invasive colorectal €anc€r. Clinical features: . . Onset early puberty Polyps ofthe colon ultimately change into adenocarcinoma by the fourth decade of life . Abnormality ofthe retina ofthe eye . Development olmultiple epidermal cysts usually on face, scalp, and extremities The oral findings ofGardner's syndrome include: . . Vultiple impacted and supernumerary teeth \4ultiple jaw osteomas which give a "cotton-wool" appearance to the These osteomas appear as dense, well-circumscribed radiopacities. Osteomas most olten dev- eiop first rvithin the angle ofthe mandible . *** \4ultiple odontomas \\rhen Gardner's syndrome is suspected based on oral findings, the patient should be referred to a gastroenterologist for consultation. Note: Multiple desmoid tumors t.tibt onatosis) and epidermoid cysts ofthe skin are also characteristic of the disease. Remember: Multiple impacted and supemumerary teeth are also seen in Cleidocranial dl splasia.

ESR rise with all of the following EXCEPT Inflammation Administration of hydroconisone Necrosis Suppuration Pregnancy

Administration of hydroconisone The ESR is the rate at which red blood cells settle out in a tube of unclotted blood, ex- pressed in millimeters per hour Blood is collected in an anticoagulant and allowed to sed- iment in a calibrated glass column. At the end ofone hour, the lab techniciar measures the distance the erlthrocytes have fallen in the tube. Elevated sedimentation rates are not spe- cific for any disorder but indicate the presence of inflammation. Inflammation causes an alteration ofthe blood proteins which makes the red blood cells aggregate, becoming heavier than normal. The speed with which they fall to the bottom ofthe tube corresponds to the degree of inflammation. ESR rises during: . . . . lnflammation Tissue degeneration Suppuration Necrosis l\Jote: Certain non-inflammatory conditions, such as pregnancy, are also categorized by hish sedimentation rates.

You have a patient that says he has an overactive thyroid. You ask about his meds and he says he doesnt remember what he takes now but that he used to take methimazole. He stopped becase it he claims it tore up his gums and roof of mouth. Which condition of the blood is most commonly caused as a reaction to medication tht could haave caused these symptoms? Thrombocytopenic purpura Agranulocytosis Sickle-cell anemia Peutz-Jeghers syndrome

Agranulocytosis Agranulocytosis is an abnormal condition of the blood, characterized by a severe reduction in the number ofgranulocytes (particL arly neutroprilr./. Note: It may also be caused by the antithyroid drugs zole). Clinical features: . . . Sudden onset of high feveq chills, fi.e., propylthiouracil, methimazole, jaundice, Oral infection with rapid periodontal destruction Oral ulcers and gingival bleeding weakness and sore throat The most characteristic feature of this condition is the presence of infection, particu- larly in the oral cavity. The signs and symptoms develop very rapidly, usually within a fev days. and death may occur soon afterward. The oral lesions are an important phase ofthe clinical aspects ofagranulocytosis. They ap- pear as necrotizing ulcerations of the oral mucosa, particularly the gingiva and palate. These lesions appear as ragged necrotic ulcers covered by a gray membrane. One impor- ranr aspect is that there is littl€ or no apparent inflammatory cell infiltration around the lesions. Histologically, this is pathognomonic of agranulocytosis. \ote: Cl clic n€utropenia is an unusual form ofagranulocytosis. These patients typically erhibit severe gingivitis. The severe ulcerations usually seen in agranulocytosis usually dLi not occlrr

Hypophosphatasia is a genetic metabolic disorder of bone mineralization caused by a deficiency in: Acid phosphatase Vitamin K Alkaline phosphatase Phosphorus

Alkaline phosphatase Hr pophosphatasia is an inherited metabolic (chemical) bone disease that results from low lerels ofan enzyme called alkaline phosphatase. This enzyme is essential to the calcifica- tion ofbone tissue. The severity of hypophosphatasia is remarkably variable from patient to patient. Some patients have blue sclera that resembles ost€og€n€sis imperfecta. There may be delbrmity ofthe arms, legs and chest. Frequent bouts ofpneumonia can occur as well as rccurrent tiactures. J h pes of h) pophosphatasia: . \eonatal: severe manifestations, respiratory failure, marked hypocalcification of the skeletal structures . Infantile: h-vpercalcemia, premature loss of deciduous teeth, skeletal malformations, failure to grorv . Childhood: short stature. frontal bossing, usually normal calcium and phosphate levels . Odontoh! pophosphatasia: children and adults rvho have only dental problems Important: The premature loss of teeth in children and adults is usually characteristic. These teeth also exhibit hypocalcification. Radiographically, the teeth display enlarged pulp chambers and pulp canals, deficient root development as well as alveolar bone loss. Rememtrer: Patient's with Paget's disease also have high levels of serum alkaline phos- phatase.

Name the malignant, epithelial cell tumor thrt chancteristically begins as a papule and enlarges peripherally, developing a central crater that erodes, crusts and bleeds. An example ofthis tumor is shown below.

Basal cell carchoma Basal cell carchoma is a common low-grade skin cancer that rarely metastasizes, but the local invasion by direct extension destroys the underlying and adjacent tissue. lt frequently develops on the exposed surfaces of the skin, face and scalp in middle-aged or elderly persons, it is very rare in mucosa. The primary cause ofthe cancer is excessive exposure ofthe sun or to x-rays. Clinical features: . Presents as a nonhealing, indurated chronic ulcer . . . Males are more affected than females 4th decade of life or older Located primarily on sun exposed areas ofhead and neck with the nose being the most common srte Risk factors: childhood sun exposure, blistering sunbums, fair skin, blue eyes, blonde or red hair. Important: It is locally destructive and metastases are exceptionally rar€. The treatment for basal cell carcinoma is eradication ofthe lesion, often by electrodessication or cryotherapy. Recurrence is unconmon ifproperly treated. r'-ote: MOHS micrographic surgery is the most effective and state-of-the-art procedure for skin cancer today, which offers the highest potential for recovery - cancer has been previously treated by another method and recurred. With the MOHS technique. physicians are able to see beyond the visible disease, to precisely identify and re- moVe the entire tumor layer by layer while leaving the surrounding healthy tissue intact and unharmed. As the most exact and precise method of tumor removal, it minimizes the chance ofre-growth and lessens the potential for scarring or disfigurement.

All of the following cysts are developmental (or fissural) EXCEPT one. Nasopalatine duct (canal) cyst Nasolabial (nasoalveolar) cyst Branchiogenic cyst Median palatal cyst Median alveolar cyst

Branchiogenic cyst Nasopalatine duct (canal): "hearrshaped" radiolucency in midline, most frequent type of nonodontogenic cyst. Usually asymptomatic or may produce an elevation in the anterior part ofthe palate.Teeth are vital, Treatment is enucleation. Note: It is caused by cystification ofthe nasopalatine duct remnants. Remember: It is also known as incisive canal cyst and may be located within the nasopalatine canal or within the palatal soft tis- sues at the point ofthe opening ofthe c anal tine papilla). . Nasolabial (nasoalveolar): (in this loccrtion it is called a cyst ol the pala- is superficially located in soft tissues of the upper lip. This is an extraosseous cyst. Treatment is surgical excision. . Nledian palatal: rare, may occur any'rvhere along median palatal raphe. May produce s$ elling on palate. Treatment is enucleation. Note: Many oral pathologists now believe that this cyst represents a more postedor presentation of a nasopalatine canal cyst . . \Iedian alveolar: rare. occurs in bony alveolus between central incisors. Distinguished liom periapical cyst by the fact that adjacent teeth are vital. Treatment is enucleation.

You are consulting on a pathological case for a fellow dentist. The biopsy of the lesion shows multinucleated giant cells and perivascular collagen cuffing. After asking about the clinical signs, your colleague mentions that the young patient seems always to be "staring off into space" and that she has "puffy cheeks." The most likely diagnosis of this case is: Aneurysmal bone cyst Central Giant Cell Granuloma Tumor of hyperparathyroidism Cherubism

Cherubism Cherubism is a benign, autosomal dominant condition ofthe maxilla and mandible, usu- all-,- found in children by 5 years ofage (it olfects males 2:i). The vast majority ofcases occur in the mandible. The bony expansion is most frequently bilateral, although unilar eral involvement has been reported. The clinical appearance may vary from a barely dis- cemible posterior swelling ofa singlejaw to marked anterior and posterior expansion of both jau s. resulting in masticatory, speech, and swallowing difficulties. Intraoralh', a hard. non-tender swelling can be palpated in the affected area. There are no associated sl sterric manifestations. The deciduous dentition may be spontaneously shed prematurely, beginning as early as three years of age. There is often delayed eruption of the penna- nenr dentition uhich is olten defective with the absence ofnumerous teeth and displace- mcnt of those present. Radiographicalll', the lesions characteristically appear as multiple, well-deltned, multi- locular radiolucencies ofthejaw. The borders are distinct and divided by bony trabecu- lae. \ote: An occlusal radiograph of the maxilla may give a "soap bubble-like" picture rr rrh m.rrillrq anrrum obliteration. Histologicalll, the lesions bear a close resemblance to those seen in central giant cell granulomas. There are nlrmerous fibroblasts and rnultinucleated giant cells with promi- nent nuclei. A distinctive feature is eosinophilic perivascular cufling of collagen sur- rounding small capillaries throughout the lesion. Although this is not always present, perirascular collagen cuffing is regarded as pathognomonic for cherubism. The treatment is cautious waiting as there is spontaneous regression of the tumors at around 25 - 30 years ol age. With increase in age and size of the patient. the defotmity produced is less noticeable.

The translocation from chromosome 22 to chromosome 9 is a finding of which leukemia? Acute lymphocyttc Ierkemia (ALL) Chronic myeloid leukemia (CML) Acute myeloid leukerma (AML) Chronic lymphocytic leukemia (CLL)

Chronic myeloid leukemia (CML) Almost 90% of patients with chronic myeloid (mteloctttic, n|'elogenous, granuloc,-tic) leukemia have the philadelphia chromosome, an abnormality in which the long am of chromosome 22 is translocated. usually to chromosome 9. Radiation and carcinogenic chemicals may induce this chromosomal abnormaliry C\IL is characterized by the abnormal overgrowth ol granulocytic precursors rn^elctblqsts and promvelocvtes) in bone marrow, peripheral blood and body tissues. C\tL is most common in young and middle-aged adults and is slightly more common in men than in women: it is rare in children. The npical symptoms of CML include: . . . . . . . spongy bleeding gums t'atigue fer er \\ eight loss moderate splenomegaly joint and bone pain repeated infections \ote: Acute myeloid leukemia (AML) rs a malignant disease of the bone marrow in s hich hematopoietic precursors are arrested in an early stage of development. AML is distinguished ftom other related blood disorders by the presence of greater than 30% blasts in the blood and/or bone rnarrow. These blasts /m|eloblasts) contain Luer rods in therr cvtoDlasm.

Explain what is meant by a carcinoma ofthe oral cavity having the following TNM designation: Tl, N2, Ml

Clinical Staging of Carcinoma of the Oral Cavity . T = Size of the primary tumor - - - TX: Primary tumor can not be assessed TO: No evidence of tumor Tis: Carcinoma in situ T1: less than 2 cm in greatest diameter T2: 4 cm in greatest diameter T3: greater than 4 cm in greatest diameter . N = Regional lymph node involvement . - NX: Regional lymph nodes can not be assessed NO: No clinically palpable lymph nodes, or lymph nodes palpable but metastases not suspected Nl: Palpable homolateral lymph node(s), not fixed but metastases suspected N2: Palpable centralateral/bilateral lymph node(s), not fixed but metastases suspected N3: Palpable lymph node(s), fixed metastases suspected \I = Distant metastasis - !tX: Presence of distant metastasis can not be assessed lfo: No distant metastasis fI1: Clinical and/or radiographic evidence of metastasis other than regional ll mph nodes

In type 1 dentin dysplasia roots appear extremely short and the pulp is Normal Somewhat smaller Extremely large Completely obliterated

Completely obliterated Dentin dysplasia is another autosomal dominant trait that affects dentin. AII teeth ofboth dentitions are affected. This condition has not been associated with any systemic con- nective tissue disorder. This is a rare condition that has been subdivided into type I or radicular type and a more rare type II or coronal type: . . Type | (radicular . . cl.,-.splasia): more common type Both dentitions are normal in color and shape The teeth are generally mobile, frequently abscess and can be lost prematurely . Teeth show greater resistance to caries than do normal teeth Radiographic features: . Extremely short roots . Obliterated pulp chambers and root canals before eruption . ( Residual fragments of pulp tissue appear typically as horizontal lucencies cnevrons) . Periapical radiolucencies Type Il (coronal . . . dy.splasia) (granulomo,s or c.vst-r') around the defective roots Color of primary teeth is opalescent Color of permanent teeth is normal Coronal pulps ofpermanent teeth are r"rsually enlarged ("thi,\tle lube") a J arc lilled rvith globules of abnormal dentin Radiographic features: . Deciduous teeth are similar in appearance to type I, but permanent teeth exhibit enlarged pulp chambers . ("thistle .\bsence of neriaoical radiolucencie" nbe") in appearance

A 21-year-old male patient is home from college and came to you because his "bite seems off " A quick physical assessment seems to indicate that his chin is deviated to the right. Taking a panoramic x-ray and comparing to the previous panoramic radiographs you have in his chart, you notice that the left condylar neck seems to have elongated. What condition does this patient most likely have? Condylar agenesis Condylar hyperplasia Condylar hypoplasia Hemifacial microsorria

Condylar hyperplasia Congenital and developmental anon.ralies ofthe temporomandibularjoint, although rela- tively rare, are important to identiry early to reestablish normal midface growth centers. The more common entities include condylar agenesis, condylar hypoplasia, and condy- lar hyperplasia. Condylar agenesis is the absence of all or porlions of the coronoid process, condylar process, ramus and mandibular body. Other first and second arch abnormalities are com- monly seen. Early treatment is indicated to limit the degree of deformity, with the pri- mary objective being to re-establish the condylar growth center. This is best done with a costochondral graft with or without orthodontic surgery and facial plasric augmentation. Condylar hypoplasia may be congenital, but is usually the result of trauma or infection. The most common facial deformity is shortness of the mandible with deviation of the chin towards the affected side. Treatment ofthe child involves the placement ofa cos- tochondral graft. In the adult, treatment involves either shortening of the normal side or lengthening ofthe involved side. Both result in an acceptable cosmetic and functional result. Onhodontic therapy is necessary in all cases to establish proper occlusion. Condylar hyperplasia is an idiopathic disease characterized by a progressive, unilateral o\ergro\rth olthe mandible. The chin is deviated towards the unaffected side. Presen- tation is common in the second decade. Radiographic findings are usually a normal condlle but an elongated neck- Treatment depends on whether the condyle is still grow- ing. lts growth is occurring, condylectomy is the treatment- If growth has ceased, or- thognathic surgery is performed.

Atl of the following aid in wound healing Except one Highly vascular areas (i.e., tongue) Hyperthermia Younger age Cortisone

Cortisone Other factors that influence the rate ofhealing: . Location of the wound: wounds in an area in which there is a good vascular bed hc'al considerably more rapidly than wounds in an area which is relatively avascular . Phl sical factors: severe trauma to tissue is a deterent to rapid woLlnd healing. The local temperature in the area of a wound influences the rate of healing. ln environ rnental hyperthemia, rvound healing is accelerated; while in hypothermia, healing is dela1,ed. . Circulatory factors: anemia and dehydration have been found to delay the healing of . q ounds -{ge of patient: wounds in younger persons heal considerably more rapidly than r ounds in elderly persons .Infection: . bacterial invasion will retard healing Hormonal factors ACTH and cortisone are substances that have been known to in- terlere rvith the healing of wounds. Diabetes mellitus (in.sulin deJiciencl) is one ofthe most $ idely recognized diseases in rvhich there is significant, clinically evident. retarda- tion in repair of wounds. \utritional factors such as the amount ofprotein a patient is consuming is one ofthe most rmpofiant factors which may effect the speed ofwound healing. Hypoproteinemia has been shos'n to delay wound healing, while having a high protein diet has been shown to accelerate u ound healing. Vitamins, especially Vitamin C, have been shown to be important in proper wound healing.

Severe hypothyroidism in a child is called Dwarfism Myxedema Cretinism Acromegaly

Cretinism Hypothyroidism refers to a condition in which the amount ofthyroid hormone in the body is below normal. This is the most common form ofthyroid function abnormality, and is far more common than hyperrhyroidism. This condition is considerably more comn.ron in women than in men. The most common cause of h)?othyroidism is Hashimoto's thyroiditis. The second most common cause is the treatment of hyperthyroidism. Hypothy- roidism is characterized by pufiiness of the face and eyelids and swelling of the tongue and larynx. The skin becomes dry and rough and the hair becomes sparse. The individual has a low basal-metabolic rate and a low body temperature. The affected individuals also have poor muscle tone, low strength and get tired very easily. Mentally they are very sluggish. The treatment of hypothyroidism is straightforward and consists of adminis- tering thyroid hormone (t hyt oxin). Se\€re hypothyroidism in a child is called cretinism. Due to a lack of thyroid hormone, there is a retardation of growth and an abnormal development of bones. Mental retardation is caused by the improper development ofthe CNS. If this condition is recognized early, it can be markedly improved with the use of thyroid hormones. Note: Extreme hypothyroidism in adults is called myxedema. \ote: Dental findings in a child with hypothyroidism include an underdeyeloped mandible with an overdeveloped maxilla, enlarged tongue which may lead to maloc- clusion. delayed eruption ofteeth and deciduous teeth being retained longer.

The soft tissue, and far less common, variant of the nasopalatine canal cyst is the: Median mandibular cyst Nasolabial cyst Cyst ofthe palatine papilla Aneurysmal bone cyst

Cyst of the palatine papilla The nasopalatin€ dvct cyst (NPDC) is a developmental non-odontogenic cyst that only oc- cuIS in the anterior maxilla from the embryonic remnants ofnasopalatine ducts. lt is usually located between teeth #s 8 & 9 but can be as posterior as the mid-palate and as anterior as the incisive papilla. It is the most common non-odontogenic oral cyst of the oral caviry lt oc- curs at any age, but is most common in the fourth to sixth decade oflife and is more common in males. It is usually asymptomatic and is discovered during routine dental examination. If infected, the patient complains ofpain with swelling and even drainage; however, swelling is not common. When present, swelling is often in the incisive papilla area in smaller lesions but can be in the mid-palate in larger lesions and may be buccal and mid-palatal in very large le- sions. Radiographically it usually presents as a bilateral, well-circumscribed, round, ovoid or heart-shaped radiolucency. Tooth displacement or diverging ofthe roots ofthe central in- cisors is commonly identified. Depending on the size and clinical symptoms, treatment ranges tiom no treatment to surgical curettage ifthe cyst is infected or interlering with a prosthetic appliance. Recurence is rare and prognosis is good. Note: The cyst of th€ palatine papilla is identical to that of its intrabony counterpart. The median mandibular cyst, like the globulomaxillary cyst, was once considered a fissural c.vst. Ho$'ever, embryonic evidence rcfutes thrs (there are no epithelial lined processes).It is a \ ery rare cyst that appears in the midline ofmandible. Most of them are periapical, lateral periodontal cysts or odontogenic keratocysts. Remember: The nasolabial cyst has been called, inaccurately, nasoalyeolar cyst. This lar ter designation is inappropdate because the entity is not a true cyst ofthe maxilla. Rather, it represents a soft tissue cyst without involvement ofthe alveolus, hence the preference for the designation "nasolabial cyst." Note: Because this cyst is extraosseous, it is not likely to be seen on a radiograph.

Which of the following is associated with a decrease in caries Sjdgren's syndrome Cystic fibrosis Cerebral palsy Down syndrome

Cystic fibrosis *** Important: There is a significantly reduced caries rate in patients with cystic fi- brosis. This is probably the result of alterations in saliva and the long{erm use of an- tibiotics. Cystic fibrosis is a congenital metabolic disorder that causes the exocrine glands (vhich are glands thqt seqete.fluids into a duct) to produce abnormal s€cretions, resulting in several symptoms, the most important of which affect the digestive tract and the lungs. In some glands, such as the pancreas and those in the intestines, the secretions are thick or solid (an ext essivef;, viscous mucous/ and may block the gland completely. The mucousproducing glands in the airways ofthe lungs produce abnormal secretions that clog the air- rvays and allow bacteria to multiply. The sweat glands secrete fluids that have a high sodium and chloride content. Note: The staining olthe teeth is most likely due to the lact that patients with cystic fibrosis are usually subjected to large amounts of tetracyclines during childhood. Si mptoms of CF include: . . . . . . Poor growth despite good appetite Malabsorption and foul, bulky stools: steatorrhea Chronic bronchitis (COPD) with cough Recurrent pneumonia: respiratory infections Clubbing offingers and toes Banel-chested appearance

A healthy l9-year-old patient presents to your office for a routine exam. Taking a panoramic radiograph, you see a well-corticated, unilocular radiolucency surrounding the crown of impacted tooth #17. The lesion is asymptomatic. What is the most likely diagnosis? OKC Dentigerous cyst Cystic ameloblastoma Central ossifying fibroma

Dentigerous cyst (or follicular cyst) Clinical features: .lt . . . usually contains a crown ofan unerupted tooth Usually not clinically visible rvithout radiographs .\\\ nrptomalic, occasionally pain or swclling Lsually involve uneruptcd mandibular third molars. olher frequenl sites includc ma\illary canines. marillary third molars and mandibular second premolars Rrdiographic feature: \\'ell-circumscribed, unilocular radiolucency around crown ol'looth. See =50 in booklet Remember: I The lateral periodontal cyst may bc dcfincd as a non-kciatinized, non-inflammatory dc- \ clopmcntal cyst occurring adjacent o. lateral to thc root ofa tooth. See picture #91 in booklet :. The fissural cysts are also called developmental .,l.irtr) are non-dental in oiigin, thcy include nasoah colar. median palatal, and nasopalatine cysis. -1. -1. ltthich Th!- primordial cyst contains no calcified structures. Thc traumatic jaw (simple) bone cyst n].ay contain blood, 11uid, dcbris or be completely empt-l.', Comrnonly found in young persons, in the mandible between the caninc and ramus. The residual cyst is often found in edentulous areas. This cyst refers to a situation in which a tooth * ith a radicuiar cyst associated with it was extracted. and the socket wasn't curelted. The radicular clst persists in thejaw as a residual cyst. The gingival cyst ofthe newborn has also been designaled as the dental lamina cyst ofthe new- born or Bohn's noduels. Such cysts appear typically as multiple nodulcs along the alveolar ridge in neonales. They are due to cystification ofrcsts ofthe dcntal lamina. In the vast majority ofcascs. these cvsts degeneratc and involute or rupture into the oral cavity. Note: Similar epithelial inclu- sional cysts may occur along the midline ofthe palate 2earl9. fpala/i/te that conlains q'sts ofthe nerborn or Epstein's Thcse are ofdevelopmental origin but are not dcrivcd from odontogenic cpithelium. No treatlnent is necessarv.

All of the following can cause intrinsic staining of the teeth except: Dentinogenesis imperfecta Erythroblastosis fetalis Porphyria Fluorosis Diabetes mellitus Pulpal injury Intemal resorption Tetracyclines

Diabetes mellitus (see chart for more details) Extrinsic stains can be caused by bacteria, iron, tobacco, foods, beverages, gingival hem- orhage. restorative materials and medications. Treatment: . lIost extrinsic stains can be removed with abrasives; reduce or eliminate cause of stain . Intrinsic stains may need bleaching, esthetic restorations, or prosthetic rehabilitation

A new 6 year old pediatric patient walks into your operatory with his mother, You initial physical assessment notes a prominent foreheada and flattened nose. The patient initially seems to have no eyebrows but you later realize that the hair is just very fine and sparse. When you shake her hand and she smiles, you also notice that she is missing teeth and the ones she has are cone shaped. What is her most likely systemic condition? Piene Robin syndrome Ectodermal dysplasia Cleidocranial dysplasia Peutz-Jeghers syndrome Osteopetrosis

Ectodermal dysplasia Ectodermrl dvsplasia is an x-rinked recessive condition charactedzed by abnormar deveropmenr ofthe .i1n 3nd associated struct]lfes lhdir noi!!, and teeth, ond s$eat gldnd.rl. It involves all structures which are ::nr ed tiom rhe €ctodcrm. It affects mares more than females. common clinical findings include hvpothrichosis /./e./?d.re in hait,). ^nhidrosis ho s\\,eat gland.r, leoding to h(,at iuh )l..r,La. oligodontia konplere or partiar absenk of teeth). depresscd bridgc of nose, Iack of salivary grands l.,l the child appears much oldcr than rvhat he or shc is. There is no treatment lor the disease. hoNever ;::rrures can be fabricated for these patients. Keep in mind that they will need to be replaced periodicallv :-r r..!rmmodate the palient's jaw growth. Scc picturcs #28 and #29 in booklet Clcidocranial d]splasia is an autosomal dominant condition ofbony developmenr characterized by hy- poplasia or aplasia ofthe clavicles' cranial bossing, ocular hypertelorism, and dental abnonnalilies which ::rrlrJe retaincd primar) teeth, malaligned leeth, the presence of multiple supcrnumerarv teeth and unerupted reeth- lmportant: The dentition itself, as obsen,ed by radiographs alone, often suggests the di- :5o.rs. Sec picturc #30 in booklet Pierre Robin svndrome is an inherited disorder that presents the fblowintl in the neonate: severe micrognsthia and mfndit ular hypoplasia. se\ ere glossoptosis c/lanodontia t,r //),,, ariot rlt,fLttenent ofthe tongtrc), and high-arched or cleft pal:tte. This condition is characterized by respiratory problems. Peutz-.Icgh€rs syndrome /PJSI is a genetic condition rnarked by hyperpigmentation (r"rfltrg ofthe lips .rnd \ometlmes other parts ofthe face, hands, and feet followed by the development ofbenign polyps called arn'anomas throughout the intestines but primarily in lhe small intestine. See picture #35 in booklet osteopetrosis /a&o.a lled Albers-Se honberg disease or narble bone./iredr., is an uncommon bone con- Jiiron thal mav be inherited as an autosomal dominant /&rr--reno .!, or recessive trait .r3racteristic feature ofosteopetrosis is an abscnce ofphysiorogic bonc resorption owing to rcduced os- (eoclastic 6drle seri.n/s./. The activity' The lack ofbone resorption results in accumulation ofbone mass and manifesls itself :. ikeletal disturbances. including bone cavity occlusion. decreased hematopoietic activity, and growth re1:rrdation. Bone pain is the mosl liequent symptom. Blindness and deafness from sclerosis ofostia. anemin riom sclerosis ofbone marroq and osteomyeliris due to diminished vascularity are aiso seen. Dental find- ings include delayed enrprion. congenitally absent teeth. unerupted and marormed teeth, and enamer hy- ooDldsla,

Cleft palate occurs in the ? of embryonic life First to third week Fourth to sixth week Eighth to tenth week Twelfth to fourteenth week

Eighth to tenth week Cleft palate occurs in the eighth to tenth week of embryonic life. Isolated clefts olthe palate are more common in females. It is characterized by a fissure in the midline ofthe palate, resulting from the failure ofthe two sides to fuse during embryonic development. The most severe handicap imposed by cleft palate is an impaired mechanism preventing normal speech and swallowing. Note: It effects approximately I in 2000 births. Cleft lip results when the medial nasal proc€ss fails to fuse with the lateral portions of the maxillary process olthe first branchial arch. Fusion normally occurs during the sixth and sev€nth weeks of embryonic development. The maxillary lip is most commonly af- fected. It may be bilateral (20a/o) or lunilateral (80%). Clefts ofthe lip are more frequent in males. Lip clefi involvement is more frequent on the left sid€ than the right. Note: It effects approximately I in 1000 births, but varies with race. L Speech problems associated with both ofthe above are usually the result of \ot€< the inability ofthe soft palate to close airflow into the nasal area. 2. It is not unusual for teeth especially the lateral incisor to be missing in the cleft area. 3. More than 250 syndromes have been identified that may be associated with cleft lip and palate, or cleft palate alone. 4. Lip pits are rare anomalies that can occur in the upper lip, lower lip, or the oral commissure. Although lip pits may be seen near the oral commrssure or midline upper lip, most occur on the Iower lip and are associated with van der \lbude syndrome. This syndrome is an autosomal dominant condition with 80% to 90% penetrance consisting of lower lip pits and cleft lip and/or cleft Dalate.

An enamel defect resulting from the incomplete formation of the enamel matrix is called: Enamel hypocalcification Enamel hypoplasia Regional odontodysplasia Enamel Pearls

Enamel hypoplasia is a developmcntal def'ect in which the enamel ofthe tccth is hard in contcxt but thin and dcficient in amount. It results from incomplete formation ofthe enamel matrix with a deficiency in the cementing substance. Enamel hypoplasia a1l'ects both the deciduous and pcnna- nent teeth. It is usually due to illness or injury during tooth formation or due to a Note: Thc gcnetic lbrms ofcnamel hypoplasia arc generally considcred to be types ofamelogen- esis imperfecta. The clinical appearance ofenamel hypoplasia includes: l) the lack ofcontact between teeth, 2) the rapid breakdown ofocclusal surfaces, 3) a yellowish-brorvn stain that appears whcre the dentin is exposed. Note: lfonly one permancnt tooth is affected, it is usually caused by physical damagc or periapical infections to the primary tooth that this pernanent tooth replaced. This is known as h-vpoplasia." Remember: Enarncl hypocalcification is a hereditary dental dcfcct in which thc enamel is soft and "Turner's undercalcificd in context yct normal in quantity. It is caused by the dcfective maturation of amcloblasts ftrere r.r a defect in the mineraliz tion ofthe fonned genetic disorder matrir). The teeth are chalky in consistency, the surfaces wear down rapidly and a yellow to brown stain appears as the underlying dentin is exposcd. This condition affects both the deciduous and permanent teeth as well. See picture #9 in bookl€t \ote: Regional odontodysplasia involves the hard tissues that are derived from both epithelial (enarnel) an<l mesenchymal (dentin and cemenhnt) components of thc tooth-fonning apparatus. Thc tccth in a region or quadrant of the maxilla or mandible are affccted to thc extent that they exhibit short roots, open apical foramina' and enlarged pulp chambers' The thinness and poor mineralization quality ofthe enamel and dentin layers havc given rise to thl; term The permanent teeth are aflected more than the primary teeth "ghost teeth." , and the maxillary anteriol teeth are affccted more than other teeth. The cause is unknown, although nume.ous etiologic factors have been suggested /1.e., trauma, nutritional defcie cies, inlbcliotts). Because ofthe poor quality ofthe affected teeth, their rcmoval is usually indicated.

A 47-year-old HIV+ patient is referred frorn his physician to you because ofan exophytic growth in the maxillary left mucobuccal fold. Unable to find an odontogenic source, a biopsy was done. A starry-sky morphology was found and a dismal diagnosis of Burkitt's lymphoma was made. Which virus is thought to be responsible for this lymphoma? Herpes virus Epstein-Barr virus Cytomegalovirus Human papillomavirus

Epstein-Barr virus Burkitt's lymphoma is a high-grade, non-Hodgkin's lymphoma that is endemic in Africa and occurs only sporadically in North America. It is manifested most often as a large osteolytic lesion in the jaw (African (Non-African form). fom) or as an abdominal mass Burkitt's llmphoma is the first human cancer with strong evidence of a viral etiology. The Epstein-Barr virus (t herpes-4tpe virus) has been isolated from cultures of tumor cells and patients with Burkitt's lymphoma have high titers ofantibodies against EBV Also, an antibody against a surface antigen on the tumor cells has been demonstrated, Note: The Epstein-Ban virus is also associated with infectious mononucleosis, oral hairy leukoplakia, and nasopharyngeal carcinoma. Two forms of Burkitt's lymphoma: 1. African: younger (mean age 3), male predominance, typically involves the 2. \on-African: older (mean age I I), no sex predilection, presents most often as an ab- dominal mass. *** Both forms are histologically identical. \ote: The jaw lesions usually present as expanding intraoral masses on the palate and ringir,a. Lesions appear as soft tissue nodular masses and many are hemorrhagic. See picture #37 in booklet. Radiographically, there is a moth-eaten, poorly marginated destmction ofbone. See picture #38 in booklet

A mother brings her 2-year-old boy into the dental oflice because of a "swelling" on his alveolar ridge. Your exam reveals a smooth-surfaced bluish lesion with fluctuance where tooth #K will be erupting. The most likely diagnosis of this is: Dentigerous cyst Eruption cyst Hematoma Hemangioma

Eruption cyst An eruption cyst is essentially a soft tissue variant ofthe dentigerous cyst. It is invariably lly prim^ry but o(casionally a permanent tooth). The effects are mostly limited to the overlying gingival tissues rather than bone. associated with an erupting tooth (asaa Clinically, the lesion usually appears as a smooth-surfaced, reddish-pink or bluish-black, tluctuant. Iocalized swelling on the alveolar ridge over the crown ofan erupting primary or permanent molar tooth. The intense bluish coloq which is often characteristic, is due ro an accumulation of blood. Due to this appearance, it may be mistaken for a hemangioma or hematoma. \o treatment is necessary as the cyst often ruptures spontaneously. In a few rare cases, incision or even the removal ofthe overlying tissue may be necessitated by pain or ten- demess associated with lhe lesion.

In your ofllce, you see a 6 month old child whose lirst teeth are eruptlng and whose mother is concerned about the color. The mandibular incbicors do show a brownlsh-blue hue. You are golng to ask the mother about which of the following conditions during her pregnancy Sickle-cell anemia Erythroblastosis fetalis Patent ductus arteriosus Low-weight preterm birth

Erythroblastosis fetalis The fetus' blood is Rh-positive because the father passed along an Rh-positive trait. rvhich is a dominant trait. The mother responds to the incompatible blood by producing antibod- ies against it. Thesc antibodies cross thc placenta into the fetus'circulation. whcrc thcy af tach to and destroy thc fctus' red blood cells, leading to anenria er.r throblastosis fetalis. \ote: It can also rcsult from blood typc incompatibilities. For cxample, the mothcr may har e ty'pc O blood and thc fetus has type A or B blood. The most common form oferythroblastosis fctalis is called is ABO incompatibility, which can ran in its severity. The less common form is called Rh incompatibility, which more ottcn causes a very scvere anemia in the baby. The severity olthis condition can vary widcly. In some instanccs, the baby has no symptoms of the disease. In othcr cases, it can lcad to death of thc baby beforc or shortly after birth. lt can be treatcd in utero by intrauterine trans- t usion. \\'hcn thc child is bom, signs may include an enlarged liver or spleen, generalizcd cdcma, laundice. be This is being absorbed - and anemia. After birth, depending on the severity, a transfusion usually nccds to performed. The most severc form of this disease, Rh incompatibility, can bc prcvcntcd if thc mothcr takes a medicinc called Rhogam at certain times during and aftcr prcgnancy. Oral manifestations of crythroblastosis fetalis: . Teeth appear to havc a grccn, blue or brown hue due to thc deposition ofblood pigment in the enamel and dentin. . Enamel hypoplasia may occur. If it does, it affects thc incisal edges of the anterior teeth and the middle portion ofthe deciduous cuspid and the first molar crown.

Pituitary adenoma in a 9 year old will most likely lead to Gigantism Acromegaly Achondroplasia Dwarfism

Gigantism In over 90% of acromegaly patients, the overproduction ol GH is caused by a benign tumor of the pituitary gland, called an adenoma. Whether or not the epiphyses of the long bones have lused with the shaft is the main detenninant of whether gigantism or acromegaly will occur when there is oversecretion of growth hormone by the pituitary gland. Remember: . Gigantism: tumor prior to adolescence (non-fusion o.f epip\,ses) . Acromegaly: tumor after adolescence (fusion of epiphyes) Oral manifestations ofacromegaly and gigantisrn include: enlarged tongue, mandibular prograthism, spacing of the teeth rvhich are usually tipped to the buccal or lingual side, oNing to enlargement ofthe tongue. Roots may be lorger than normal. See picture #33 in booklet. \ote: Dn arfism (pttuitary dvtar/i) is characterized by arrested growth. Frequently these people have limbs and features not properly proportioned or formed. It is caused by undersecretion ofgfowth homone. Oral manifestations include: eruption rate and the shed- ding ofthe teeth are delayed, clinical crowns appear smaller as do the roots of'the teeth, rhe dental arch as a whole is smaller causing malocclusion and the mandible is under- der eloped.

While attempting to give in inferior alveolar nerve block, if you inject the anesthetic solution into the capsule of the parotid gland, you may cause a Bell's palsy like feeling for the patient by anesthetizing the Trigeminal nerve Glossopharyngeal nerve Hypoglossal nerve Facial nerve

Facial nerve Bell's palsy is a form offacial paralysis resulting from damage to the facial n€rve.It can strike at any age; however, it disproportionately attacks pregnant women and people who have diabetes, influenza, a cold, or some other upper respiratory ailment. Clinical signs include a unilateral paralysis ofall facial muscles with loss ofeyebrow and forehead wrinkles, drooping ofthe eyebrows, flattening ofthe nasolabial furrow, sagging of the comer of the mouth and the inability to frown or raise the eyebrows. The upper and loler lips may also be paralyzed on the side affected. .{tier its sudden onset the paralysis begins to subside within two or three weeks, and grad- ual. complete recovery occurs in over 85yo of patients. Triggering events related to Bell's palsy are acute otitis media, atmospheric pressure change. exposure to cold, ischemia of the facial nerve near the stylomastoid foramen, \lelkersson-Rosenthal syndrome, and multiple sclerosis. Note: Melkersson-Rosenthal slndrome is the term used when cheilitis occurs with facial palsy and plicated tongue.

Which of the following is a troublesome fibroblastic neoplssm that is locally aggressive and infiltrative? Peripheral fibroma Traumatic neuroma Nodular fasciitis Fibromatosis

Fibromatosis \ traumatic ncuroma is a lesion causcd by lrauma to the periphcral ncnc. In thc oral catity, thc injury rnay be nr rhr rirmr olrrauma fiom u surgical procedure such as a toolh cxlraclion, from a local ancslhctic injcclion or fionl an .r..ideni. It is usually a vcry small nodulc 1/€.\'r /,{utt 0.5 .r in didnrcter).In thc oral cavity it is most commonly sccn .! rIc mentat foramen. Il is firm, movable ard wcll cncapsulatcd. ll is rhr naurona clicits a resporsc oftcn dcscribcd as an "electric shock". Hi!tologic features: . \bundant ncnc lissuc a'rd collagcnous fibrous tissuc in haphazard arangcnrcri . . Chronic lillammalory ccll infilirate may be Sch\ ann cclls rri1l bc prcscn

A healthy patient comes into your ollice for an inltial exam. On the full mouth series of radiographs you see a radiopaque lesion periapical to tooth #19. Tooth #19 has a deep amalgam restoration with recurrent decay underneath. You can see the entire outline of the mesial root of tooth #19 - the lesion seems to stem frorn the tooth. What is the most llkely diagnosis of the lesion? Focal sclerosing osteomyelitis Cementoblastoma Cementoma Fibrocementoma

Focal sclerosing osteomyelitis Focal sclerosing osteomyelitis is a relatively common phenomenon that is believed to repre- sent a focal bony reaction to a low-gnde inflammatory stimulus. [t is usually seen at the apex of a tooth in rvhich there has been a long standing pulpitis. Note: Synonyms for focal sclerosing osteomyelitis include bony scar, condensing osteitis, and sclerotic bone. The term focal periapical osteopetrosis has also been used to descdbe the lesions associated with normal caries-free teeth. Focal sclerosing osteomyelitis may be found at any age but is typically discovered in young adults. Patients are usually asymptomatic, and most lesions are discovered on routine radi- ographic examination. A majority are found at the apices ofmandibular first molars. The pe- riapical x-ray demonstrates the pathognomonic, well-circumscribed radiopaque mass of sclerotic bone surrounding and extending below the apex ofone or both roots. The entire root outline is almost always visible, an important litature in distinguishing it from the benign ce- mentoblastoma, which radiographically, it may resemble. The tooth with this lesion may be treated or exftacted, since the pulp is infected and the infection has spread past the immediate penapical area. The sclerosing bone constituting the osteomyelitis is not attached to the tooth. and remains after the tooth is treated or removed. DifTuse sclerosing osteomyelitis represents an inflammatory reaction in the mandible or max- illa. believed to be in response to a microorganism oflow virulence. lmportant in the etiology and progression ofdiffuse sclerosing osteomyelitis is chronic periodontal disease, which appears to provide a portal ofentry lorbacte a. The condition tends to occur most ftequently in middle-aged black females. The disease is typified by a protracted chronic course with acute exacerbations of pain, swelling, and occasional drainage. Radiographically, this process is dld'ure. 9*pically affecting a large part ofthejaw The lesion is ill-defined. Treatment consists t idetermining and addressing the cause. Antibiotics are the mainstay of trcatment. Low-dose .onicosteroids have also been used with some success. Hyperbaric oxygen therapy may prove :.r be a r aluable adjunct.

All of the followlng cysts are congenital EXCEPT one. Thyroglossal duct cyst Branchial cyst Globulomaxillary cyst Dermoid cyst

Globulomaxillary cyst Thc thl roglossal duct cyst, which may arise fiom any portion of the thyroglossal duct. This cy st is therefore lound in a midline position and is usually dark in color It may bc vascular as to rcsenrblc a hemangioma. One lrequent important symptonr is hemorrhage into the mouth, resulting from thc rupture of the overlying veins. Complctc cxcision of the tracl to the base of the Iongue. frequently including a portion ofthc hyoid bone, is nccessary tbr a curc. Note: Whcn mirration ofthc thyroid gland lails and the gland rernains in thc basc ofthe tongue. it is called lin- gual thl roid or eclopic lingual thyroid. Lingual thyroids are lour tjmes more comnon in females than in rrales. . The branchial /.1eli/ cyst or cervical lyrnphoepithelial cyst is located in the latcral portion ofthe ncck. usuallv anterior to the sternocleidomastoid muscle. Thcy may also appear in the submand- rbLrlar area. adjaccnt to the parotid gland. or around lhc stcmocleidomastoid muscle. The majorily rr: lhese t-vpes arise ftom remnants ofthe second branchial cleft or pouch. This cyst is lincd with .rlified and slratified squamous epithelium and conlains a l11ilky or mucoid fluid. The treatment aon5isrs ofconlplele surgical excision. Note: The branchial cyst has an intraoral countcrpart known a: Ihe lt mphoepithelial cyst. The floor ofthe moulh is the most common site lor thcse lcsions, foll- o\\ ed bv the tongue . Thc dermoid cyst is rclativcly uncommon in the oral cavity. This cyst frequcntly contains hair, \ebaceous and sweat glands, as well as tooth structures. The most common site is thc midline of the floor ofthe mouth ifabove the mylohyoid muscle.lt appcars as a mass in the upper neck if belol\ the mylohyoid muscle. The treatment is the surgical removal ofthe entirc tulnor Important: Globulomaxillary cysts are thought to dcvclop fiom epithelial rcmnants remaining tbl- los ingjoining ofthc globular portion ofmcdian nasal process with thc maxillary process. They arc characterized by a large "pear-shaped" radiolucency between the maxillary lateral incisor and cuspid. Thcy arc asymptomatic, all regional teeth are vital. They tcnd to cause divergence ofthc roots. However, thcrc is considerable controversy as to whether this cyst actually exists. Many of these are, in reality apical cysts associated with non-vital lateral incisors. Olien thcsc apical cysts havc a tcndcncy to cxtcnd betwccn thc two tccth to simulatc a globulomaxillary cyst.

A dentist isoften consulted first by a ptient with pernicious anemia for relief of Denuded gingiva Glossitis Edematous buccal mucosa Severe gingivitis

Glossitis Pernicious anemia is a relatively common, chronic, progressive, megaloblastic anemia. It is caused by the lack of secretion of the intrinsic factor in normal gastric lactor is necessary lor adequate absorption of vitamin 81.,, which is necessary for the maturation of erythrocytes. As a result, they produce fewer erythrocytes than normal. The onset of pemicious anemia usually is insidious and vague. As the condition pro- gresses, there will be a sore, painful ton g.ue juice. (atrophic glossilr.r/, angular cheilitis, a tingling numbness of the extremities. difficulty swallowing (od,vnophagia). See picture #20 in booklet (d,,-sphagia), painful swallowing A Schilling 24-hour urine test is done to evaluate whether vitamin B by the body and is most commonly used to evaluate patients for pemicious anemia. 1.' Remember: Thalassemia major and minor are hernolytic an€mias that result from a ge- netic defect. Both are characterized bv a low level of ervthrocvtes and abnomal hemo- globin. Oral manifestat ions of thalassemia: . . Oral mucosa may exhibit the characteristic anemic pallor Flaring of the rnaxillary anterior teeth with malocclusion

A 53-year-old patient comes to your office and notes that sometimes when he swallows, he gets a sharp 'jolt" on the right side of his throat. He says the pain is severe and he can even feel it in his ear. The most likely diagnosis is: Postherpetic neuralgia Orolingual paresthesia Frey's syndrome Glossopharyngeal neuralgia

Glossopharyngeal neuralgia Glossopharyngeal neuralgia refers to pain similar to that of trigeminal neuralgia, which arises frorn the glossopharyngeal newe (CN IX).lt is not as common as trigeminal neu- ralgia. but the pain may be as severe when it does occur. It occurs in both sexes, in mid- dle-aged or olcler persons and is described as sharp, jabbing, electric, or shock-like pain located deep in the throat on one side. lt is generally located near the tonsil although the pajn may extend deep into the ear. It is usually tiggered by swallowing or cherving. It is almost ahvays unilateral. Frel 's sl ndrome (also called the auriculotemporal syndrome) ts an unusual phenome- non. * hich arises as a result of damage to the auriculotemporal nerve and subsequent reinnen ation of the sweat glands by parasympathetic salivary hbers.The syndrome fol- lo* s some surgical operation such as removal ofa parotid tumor or ramus ofthe mandible, trr an infection of the parotid that has damaged the auriculotemporal newe (a branch of I ---l /. lmportant: Gustatory sweating is the chiefsymptom ofFrey's syndrome. The patient rypically exhibits flushing and sweating of the involved side of the face during eating. This syndrome is not a common condition. Postherpetic neuralgia is a persistent burning, aching, itching and hyperesthesia along distribution of a cutaneous nerve following an attack ol herpes zoster. It may last for a ferv rveeks or many months. lnvolvement of the facial nerve and geniculate ganglion produces the Ramsey Hunt Syndrome, which is characterized by facial paralysis and otalpia karache).

Epithelioid cells and giant cells are derived from macrophrges and are important in the development of: Granulomatous inflammation Acute inflammation Subacute inflammation Initial inflammation

Granulomatous inflammation Granulomatous inflammation is a subtype of chronic inflammation and is characterized by granulomas, which are nodular collections of specialized macrophages referred to as epithelioid cells. A rirn of lymphocytes usually sunounds granulomas. Granulomas are produced by multinucleat€d giant cells (Langerhans giant cells and./brcign bod1, giant cells). All the other cell types characteristic of chronic inflammation, including plasma cells, eosinophils, and fibroblasts, may also be associated with granulomas. Note: Granulomatous inflarnrnation is characteristically associated with areas of caseous necrosis produced by infectious agents, particularly Mycobacterium tuberculosis, . Tuberculosis: caused by the inhalation of Mycobacterium tuberculosis. Oral non- healing chronic ulcers lollow lung infection. Important: Characterized by caseating granufomas with multinucleated giant cells (Langerhans giont cells andJbreign body giqnt cells). Etiologic agents associated with granulomatous inflammation; . . . . Infectious agents - TB and leprosy, which are mycobacterial diseases Fungal infections Spirochetes (blastomyco,sis, histoplasmosis, qnd (Treponema pallidum, which causes syphilis) Cat scratch disease Foreign material (caused coccidioidom)'cosis) h; an unnamecl gyam-negdive organisnt) (e.g., suture or talc) Sarcoidosis (unknown etiology; it is non-necrotizing) Crohn's disease (il ls non-caseating)

The most acute and severe type of hyperthyroidism is Toxic nodular goiter Graves disease Hashimoto's disease Addison's disease

Graves disease The rerm hr..perthyroidism refers to any condition in which there is too much thyroid hormone tln rorr'r, in the body. This most commonly results fron a generalized overactivity ofthe entire ihlroid gland. a condition also known as diffuse toxic goiter or Graves'disease. Altematively, one or nrore nodules or lumps in the thyroid may become overactive, a condition known as toxic nodular goiter or Plummer's disease, The primary role ofthyroxin is to stimulate cellular me- rabolism. grorvth and differentiation of all tissues. In excess, it leads to high basal metabolism, tilrgue. \! eight loss, excitability. elevated temperature and generalized osteoporosis. Oral manifestations are not too remarkable, but ifthe disturbance begins in the early years of life, the eruption of the teeth and the prernature loss of the deciduous dentition are common dndings. F.en'rature T! pes of hi perthJ-roidisml. (Note: Besic symptoms of htperth,'-/oidisn dre present as h,ell as lJdi|io aI \nptonts). . . Gra\'es'disease: is the most common form, occurs most frequently in women under 50 - - goiter knlarged thyoid which moy catrse a bulge in the netkl erophthalmos is common thickened skin over the shin area Plummer's disease (toxi. nodular goiler; afibcts both genders usually over 50 Erophthalmos is rare Ofien uni-system, may present with only cardiac disease lmportant: Thyroiditis causes temporary hypcrthyroidism, usually followcd with hypoth- roidism. Thyroiditis is an inflammation ofthe thyroid gland. Therc are three main types of thyroiditis: l . . . Hashimoto's thyroiditis Subacute granulomatous thyroiditis Silent lymphocytic thyroiditis

An 8 year-old girl who looks like she is 14 years old comes with her father into your ollice. Her father states during the health history that she has Mccune-Albright syndrome. Which of the following would you NOT EXPECT the patient to have? Heart disease Polyostotic fibrous dysplasia Cafe-auJait spots Endocrine dysfunction

Heart disease \lbright's syndrome (also cqlled McCtne-Albright s-vtdrcrre/ is the most severe form of poll-ostotic librous dysplasia. It affects young people (males andJbmales equally). It is characterized by Caf€-auJait spots on the skin, and endocrine abnormalities most conunon ol u,hich is precocious sexual development infemales). The extent to which each ofthese problems exist in those with the syndrome is quite variable. The hallmark of Al- bright's syndrome is premature puberty in the female. Early sexual development in the nrale is less comnon than the female. Clinical features: . . Early childhood \{ultiple, slow-growing, painless expansile bone lesions confined to the craniofacial area or throughout the skeleton . endocrine manifestations; in females often sexual precocit-v .lrregular . . *** Disfiguring shaped Cafe-aulait spots on the torso and sometimes intraorally Increased level ofserum alkaline phosphatase Pathologic fractures are frequently associated with this syndrome. There is no specific treatment for this syndrome. Drugs that inhibit estrogen production, such as testolactone. have been tded with some success. Important: An additional complication is the malignant transformation potential of both the polyostotic (nainly) and monostotic fibrous dysplasia into osteosarcomas.

Your friend went oyster fishing and got sick. When he went to the ER they told him he had hepatitis. Which one is most likely? Hepatitis A Hepatitis B Hepatitis C

Hepatitis A A is a highly contagious infectious disease involving the liver It is usually transrnined by dre fecal-oral route. However, it may also be transmitted parenterally. tlepati- ris -{ usually rcsults liom ingestion ofcontaminated food. milk. or water. Many outbreaks of tl'ris tvpe are traced to ingestion ofseafood from polluted water. It is caused by an RNA enteroYirus. [t most often occurs in young adults. The initial symptoms (bver malaise, ,tbdoninal -1-6 l eeks. pain. anorexia, jaundice.)of l4tlLl- Hepatitis A appear after an incubation period of Darnage to the liver cells. also results in increased serum levels of enzymes, such as transaminases, nonnally active in liver cells. The detection ofincreased serum levels of rhese enzlrnes is used in diagnosing this disease. In most cases ofHepatitis A, the infecrion is selt'-limiting and recovery occurs within 4 months. Oral complications: The only oral complication associated with hepatitis is the poten- tial for abnormal bleeding in cases ofsignificant liver damage. Ifsurgery is required, it is advisable to check with the patient's physician prior to scheduling the sLrrgery. I . The presence of surface antigen (A \otei the patient is potentially infectious for Hepatitis or B) in a patient's serum indicates that (carrier state). ...,.:-,,,a:. 2. Hepatitis viruses are very heat-resistant (morc so than the AIDS virus). 3. Autoclaving properly will kill these viruses.

Which type of Hepatitis is only found in those with acute or chronic Hepatitis B? Hepatitis A Hepatitis C Hepatitis D Hepatitis E

Hepatitis D The Hepatitis B virus (HB V) rs a double-stranded DNA virus with worldrvide distribution, transmitted by parenteral and sexual contact. Risk factors include multiple sexual partners, intravenous drug abuse, and rcccipt ofblood products. Its incubation pcriod is 40 to 100 days, and it can be recovered from all body fluids, most importantly, blood, brcast milk, and amniotic fluid. The signs and symptoms are similar to hepatitis A (fever abdominal pain, nausea, etc.) but there is a longer incubation p€riod f6-8 ueefs). The symptoms arc slorver in devcloping but are ofa longer duration. Most patients rccover fully, however, some de- velop chronic liver disease. Previously, viral hepatitis that was not caused by the type A or type B virus was callcd "non- A. non-B hcpatitis." Rccently three more viruses havc becn identilied that causc some of these non-A. non-B infections. These new Yiruses includ€: . Hepatitis C: is a serum hepatitis that is caused by a virus antigenically different from Hepatitis viruscs A and B. Most cases olpost-transfusion hepatitides are ofthis type. It is usually much milder than A or B but is otherwise clinically indistinguishablc from thcm. Thcrc is a higher incidence of chronic disease (chroni< lrcp41llrt, cirrhosis and hepatocellular carcinoma. Note; Hepatitis C is now the most common reason for liver trirnsplantation in the U.S. . Hepatitis D is found only in patients with acute or chronic episodes of Hepatitis B, and it makes the Hcpatitis B infcction morc sevcrc. Drug addicts are at relatively high . Hepatitis E: is transmittcd cntcrically, much like Hcpatitis A. It causes occasional epi- d.'nics similar to those caused by Hepatitis A. So far these epidemics have occurred only in undcrdcvcloned countries.

The clinical features of the primary form of which disease is classically described as "stones, bones, groans, and moans?" Paget's disease Hypophosphatasia Hyperparathyroidism Hyperthyroidism

Hyperparathyroidism H) perparathyroidism (von Recklinghausen's disease of bone) is a tnctabolic disorder in which the parath.lroid glands produce too nruch pamthyroid honnonc. Too much parathyroid horrnone causes loo much calcium to be released from bon€. It may be caused by a f'unctioning parathyroid tunor or conlpensatory parathyroid hyperplasia due to renal lailure, malabsorption, or vitamin D defi- crency. Thcrc is a ferralc prcdilection and it affccts middlc agcd adults. Thc symptoms include: Ioss ol'appetite, increasing thirst, lrequent urination, Jethargy and fatigue, muscle weakness, pain and constipation. Important: pathologic fraettre (due lo the marked rcsorplion of borc) mal bc thc first symptom ofthe disorder Intraorally, thcrc is diffuse bonc loss causing malocclu- sron and shifting ofthc teeth. The chie I radiographic finding is the appcarance ofwell-defined cystic radiolucencies ofthejaw, *hich mav be unilocular or nultilocular Partial loss ofthe lamina dura is seen around thc tccth. See picture #34 in booklet lmportant: Histologically, multinucleated giant cells are scattered within a dclicate fibrocellular 'iroma. Accumulalions ofhcmosiderin and extravasated red blood cells also rc present. As a result, thc ussue-i appear rcddish-brown, accounting for the term rcall) idcntical to qentral giant cell granulomas. "brown tumor." These lesions arc miuroscop- Thc discasc spectrum ofprimary hyperparathyroidism rangcs from asynptomatic cases nosetl fion routine senon calcium determiralrotrs/, to severe cases oflcthargy and occasional coma. Earl) synrptoms include faligue. weakness, nausca, anorexia, polyuria, thirst, depression, and constipation. Frequcntly, bone pain and headaches are present. There are several clinical f'caturcs ass- bones, groans, and moans." Lcsions ofthe kidneys, skclctal system, CI tract, and nervous systcm are responsible lor this syn- ociated primary hyperparathyroidism, classically described as "stones, drome complex. The rcnal component includes the prcsenc€ ol'renal calculi. Management of primary hyperparathyroidism is aimed at eliminating the parathyroid pathology. Surgery is the treatmcnt ofchoice.

An emergency patient walks into your office with swelling of the left submandibular space. He says his lower left molar recently 'broke down" and has been very painful especially when something cold hits it or when he chews down on it. What is the most likely etiology of this swelling. Orthodontics Trauma Infection of the pulp of the tooth Periodontal disease

Infection of the pulp of the tooth This infection follows the carious involvement ofthe tooth. The cellular debris and/or rnfection rvhich caused the tooth pulp to die, slowly filters out of the tip of the root and produces an inflanmatory reaction around the root tip \ote: ,\ periapical abscess can also occur after traumatic injury to a tooth, which results in necroiis of the pulp, and in cases ol irritation of the periapical tissues, either by mechanical manipulation or by the application of chemicals in endodontic Clinical features: . . If acute, presents as an abscess: See picture #26 in booklet - Tooth is extremely painful to percussion - May feel slightly extruded from its socket Tooth will exhibit mobilitY procedures' If chronic, presents as a granuloma or cyst. There are usually no clinical features of svmptoms Radiographic features: . If acut€, only a slight thickening ofthe periodontal membrane is noticeable .If chronic (granuloma or q,st), there will usually be a radiolucent area at the apex of the involved tooth. See picture #27 in booklet Treatment: Establish drainage either by opening the pulp chamber or extracting the tooth. Note: If a periapical abscess is not treated, it can lead to serious complications such as osteomvelitis. cellulitis and bacteremia.

The central giant cell granuloma Is a common lesion and appears more frequently than does the peripheral giant cell granuloma Is found predominantly in children and young adults . Affects males more than females . Is present almost exclusively in the small bones of the hands and feet

Is found predominantly in children and young adults . The central giant cell granuloma is a benign process that occurs almost etclusively within the bones. The tumor typically prescnts as a solitary, radioluccnt lesion ofthc mandible or maxilla- It is an uncommon lesion and occurs less frequently lhan does the peripheral giant cell granuloma. It is loL)nd prcdominantly in children and young adults. Fcmalcs are atltcted lnorc frcquently than males. Lcsions occurmorc frequenlly in thc mandible than in themaxilla. These lcsions tend to involvc thcjaws entarior to thc molar teeth. Thc ccntral giant coll graiuloma typically produces a painlcss cxpansion or .\\ elling ofthc affected (-entral lre jaw giant cell granuloma is classified into aggressive and non-aggressive types; the aggressrvc tends to occur in younger patients and is known to cause disfiguration, especially after surgcry. Radiographically, it consists ofa multilocular or, less frcqucntly. unilocular radioluccncy ofbonc. The r::rgins of the Iesion are relatively well demarcatcd. often presenting a scallopcd border. Roots of the i3.th nlay br displaced and. less commonly, rcsorbed. See picture #,ll in booklet Important: Thc lcsion is composed ola proliferation ofspindled libroblasts in a slroma contain ing \ ari.rb.c anlounts of collagen. N{ultinucleated giant cells are ptesent lhtoughout the conneclivc tissuc \otei Thc differential diagnosis includes amcloblastoma. odontogenic mlxoma, odontogenic kerato- .l !t. and aneurysmal bone cyst. Thc usual treatment is surgcry however. treatment altematives to surgery have emcrged with success_ 1u1 results ranging from stcroid injections to calcitonin injcctions or nasal spray to interferon alpha-2a injections . \\,hich are administered 2-J times pcr rvcck for several months. . L The microscopic appearance ofccntral giant ccll granulona is vimrally identical to ihe \otei' giant cell lesion /B/?rtri /rrrol/ associated lvith hyperparath] roidism. However. blood . . --' tests \r'ill shotv an increase in serum calcium and alkaline phosphatase and a decrcasc in serum phosphorus in hypcrparathyroidism. 2. The giant cell tumor of bone may present with similar clinical and mjcroscopic features. However, the Siant cell tunor is regardcd as rare in thejaws in comparison to the cenffal giant ccll granuloma. They occur most frequently in long bones. See picture #96 in booklet

All of the following statements concerning amelogenesis inperfecta are true EXCEPT one. Which one is the EXCEPTION It is an inherited condition which is transmitted as a dominant trait Because ofthe enamel malformation, the teeth of individuals with amelogenesis imperfecta are often discolored, sensitive to temperature changes, and painful to brush It only affects the permanent teeth It causes the enamel of the teeth to be soft and thin The teeth appear yellow. because the dentin is visible through the thin enamel The teeth are easily damaged and susceptible to decay

It only affects the permanent teeth -{melogenesis imp erlecta (AI) is a relatively rare group ofinherited disorders characterized bl abnormal enamel formation. The term amelogenesis imperfecta is reserved for hereditary defects ofenamel that are not associated with defects in other parts ofthe body or other health problems. lt is a hereditary ectod€rmal defect, unlike dentinogenesis imperfecta which is a hereditary rnesodemral defect. The AI enamel defects are highly variable and include abnomralities that are classified as hy- poplastic defect in amount ofenamel),hypomaturatron ol'euottel crystallites), andhypocalcified (defect in fnal grov'tlt and mqhration (defect in initial crystallite Jbnnation followed by dete. ti\ e gt'ovlh). The enamel in both the hypomaturation and hypocalcified Al is not mineralized to the level of normal enamel and can be described as hypomineralized Al can be intrerited as an x-linked, autosomal recessive (,4RJ, or autosomal dominant The color ofthe enamel ranges from white opaque to yellow to brown, it is reduced in volume and pitled. Contact points between teeth are often open and occlusal surfaces and incisal edges frequently are severely abraded. Although the enamel are soft and irregular, there is no in- crease jn caries mte. Note: Open bite is a common clinical finding. The radiographic findings are fiequently distinctive and pathognomonic. when the enamel is totally absent, the radiographic appearance makes the diagnosis obvious. When some enamel is present, thin radiopaque coverings on the proximal surfaces ofthe teeth are noted. When the anatonic crown forms are normal or nearly normal, the softness ofthe defective enamel may not be easily distinguished from the dentin. ln all cases, howevel the dentin' mentum are unaffected by the disease process itself(arlr*e dentinogenesis inperkdq). See picture #8 and #10 in booklet Exception: Amelogenesis imperfecta will only show pulp obliteration if there is advanced abrasion with secondary dentin formation. Other than cosmetic restoration, no treatment is necessary.

While doing a dental mission trip in ethiopia, you notice purplish-brown nodules on the hard palate of a 32-year-old female patient. You notice more of these spots on her arms and legs. Given that HIV infection has an almost 5% prevalence in urban Addis Ababa, what neoplasm are you suspicious of? Nicotinic stomatitis Hemangioma Kaposi's sarcoma Leukemia

Kaposi's sarcoma Kaposi's sarcoma is a malignant ncoplasm ofcndothelial cells. It is characterized by abnormal vascular profiferation (it f629l,,/. Thosc lo 709'0), \Nith as is o concer oftl1e lining of blood t'esselsi. It occurs on multiple sites, cspecially thc lo\ er extremities. lnitial lcsions are srnall, red papules, which enlargc and fuse to form purplc_to-broq'n. ipLrngl nodules. It spreads to lymph nodes and intemal organs. Note; Human herpes virus 8 has an eti- oloclc role. Kaposi s sarcoma is a uniquc form of angiosarcoma and is the most common cancer associated *ith predilection for the palate. Tbree different clinical patterns: AIDS patients and has a . . ' Clas\ic lfediterranean type Endcmic African type I mmunodeficiency rJpc (AIDS-tetuted). \ote: Inrraorally. the hard palate is the most common localion, followcd by the gingiva and buccal mu- R€memberAIDS is caused by the RNA refovirus, HIV (d&o tdllel HTLV-lll). The HIV infection is acqnircd with IV drrrg use as we1) as by scxual conlact (homosexual and heterosexnal) and contaminated blood Droducts.

EBV is associated with all EXCEPT Nasopharyngeal carcinoma Oral hairy leukoplakia Burkitt's lymphoma Koplik spots

Koplik spots The Epstein-Barr virus (EBV) is a member olthe herpes virus group. It causes infectious mononucleosis and has been associated with the subsequent development of two forms of cancer: Burkitt's lymphoma and nasopharyngeal carcinoma. EBV is also associ- ated rr ith hairy leukoplakia, an opportunistic infection resulting in white patches ofthe Iareral tongue. Hairy leukoplakia is a nonmalignant lesion seen almost exclusively in AIDS patients. The virus specifically infects Blymphocytes and some epithelial cells. Ir is associated rvith the production of atypical lymphocytes and lgM heterophile an- tibodies identified by the heterophile test (also callecl the mononucleosis spot lest). This antibody eventually appears in the serum of more than 80% ofthe patients with ir- teL-rious mononucleosis, hence it is highly diagnostic of the disease l.There are no specific oral manifestations ol infectious mononucleosis, al- \otes though secondary Iesions do occur Remember: 2. Neck srvellings are cl.raracteristic ofinfectious mononucleosis, Hodgkin's dis- ease and tuberculosis. . Rubella viruses cause German measles (rubella),which present with a characteristic 'ash (flat, pink spots on theface and then spreads to other bodl,parts). . Paramyxoviruses can cause measles (rubeola) and mnmps. Rubeola is characterized by fever. malaise and by the formation of Koplik's spots in the oral cavity. These spots are small. bluish-white lesions sunounded by a red ring. They cannot be wiped offand occur opposite the molars. Mumps cause enlargement of the parotid glands. Serious complications include deafness in children and orchtlis (in/lammation ol the testis) in males past puberty.

An 8 year-old chlld has complained of pain on the left side of his head for 5 weeks. There are no abnomal findlngs on physical examination. A panoramic radiograph reveals multiple radiolucent lesions on the left side of the moxilla. The lesions give ahe appearance of teeth that are "floating in space." The lesions are sharply circumscribed, with a punched-out appearance. Which of the following is the most likely diagnosis? Langerhans cell disease Hyperparathyroidism Cherubism Paget's disease

Langerhans cell disease ldiopathic histiocytosis or Langerhans cell disease, also formcrly known as histiocylosis X, is a disorder charactcrized by a prolifemtion of cells exhibiting phenotypic charactcristics of Langerhans cells, The clinical manifestations of this process range from solitary or multiplc bone lesions to disseminated visccral, skin, and bonc lcsions. Traditionally, idiopathic histiocytosis has been used to encotrlpass thrcc disorders: eosinophilic granuloma, Hand-Schullcr-Christian syndromc, and Lellcrcr-Si$,e disease. . Eosinophilic granuloma (cltronic tiplc bonc lcsions only . Hand-Schuller-Christian syndrome loculized Jbnn) (chronic has refened to patients with soJitary or rnul- listelnitvl?d lbrn.) has represented a specific clinical triad of lytic bone lesions, exophthalmos, and diabetes insipidus. Many ol thesc paticnts also cxhibit lymphadenopathy, dermatitis, splenornegaly, or hepatomcgaly. . Letterer-Silve disease (.rcute dissemitvted fonn) has been charactcrizcd by a rapidly progrcs- sir e. usually latal, clinical coursc. Widcsprcad organ, bonc, and skin involvement by the prolif- Lrdti\e lrocc:'s rn Inl'anls has been the comnton presenlalion. ldiopathic histiocytosis is generally regarded as a condition ofchildren and young adults. Oral changes may bc the initial presentation in all fonns ofthe disorder. Tenderness, pain and srvelling arc liequcnt patient complaints. Loosening ofthe teeth in thc arca ofthe affected alvcolar bone is a conlnlon occulrcncc. Thc gingival tissues are frequently inflamed, hypcrplastic, and ulceraled.The ja\\ s mal exhibit solitary or multiple radiolucent lesions. The lesions frcqucntly ali'ect the alveolar bonc. rcsu)ting in the appearance of teeth that arc "floating in space." Bone lcsions rvitb a .h:rrpiy- circumscribcd, punched-out appearance may also occu. in the central aspects of the :randiblc or naxilla. \licroscopicall!, eosinophils are mixed with thc tumor Langcrhans cells. some ofwhich arc mult- rnuc lci e d. Prognosis is Very good when the disease is localized, however, the disseminated form is usually :...te1.

While in the OR on a general surgery rotation, a 3 month old is brought in with a large (20cm) fluid-filled mass on her neck. The diagnosis is a cystic hygroma (hygroma coli).This lesion is under which umbrella of lesions, which also ' conntains enlarged tissue on the posterior and lateral border of the tongue? Angiomas Lymphangiomas Schwanaomas Fibrosarcomas

Lymphangiomas Lymphangiomas are benign bamartomas of lymphatic channels that develop early in life $ ith no sex predilection. They may occur on the skin or mucous membrane. ln addition to the tongue, they occur commonly on the lips and labial mucosa. Clinical features: . . ' . . . Raised, diffuse. bubbly nodules or vesicles Range in color from clear to pink. dark red, brown or black .As)'mptomatic Soft. fhrctuant \ aries in size Usuall.v painless Histologic features: four types of lymphangiomas: . Ll mphangioma simplex (capillory lymphangiono) - composed of small, thin-walled lvmphatics . Calernous lymphangioma ing adventitia . cl stic lymphangiom? - (cyslic comprised ofdilated lymphatic vessels with surround- hlgroma or hygroma col, - consisting ofhuge, macroscopic lymphatic spaces with surounding fibrovascular tissues and smooth muscle .Benign lymphangioendothelioma facquired progressive lltnphangiont) lymphatic channels appear to be dissecting through dense collagenous bundles Important: Lymphangiomas do not undergo malignant change. Some lymphangiomas, es- pecially congenital types, regress spontaneously during childhood. Aspiration is manda- tory before surgical excision ola lyn.rphangioma to prevent complications associated with the similar-appearing hemangioma

The most common cause of xerostomia is Hereditary Medications Tooth decay Mouth breathing

Medications Xerostomia (dry nouth) is not a disease, however, it can be a symptom of certain dis- eases. Many times xerostomia is caused by failure ofthe salivary glands to function nor- mally, but the sensation can also occur in people with normal salivary glands. Xerostomia can cause health problems by affecting nutrition, as rvell as psychological health. At its most extreme, it can lead to rampant tooth decay and periodontal disease. Perhaps the most prevalent cause of xerostomia is medication. The main culprits are antihistamines, antidepressants, anticholinergics (e.g., atropine and scopolamine), anorexiants, antihypertensives, antipsychotics azine), anti-P arl<tnson agents, diuretics and sedatives. (e.g., chlorpromazine and prochlorper- The most common disease causing xerostomia is Sjiigren's syndrome a chronic inflammatory autoimmune disease that occurs predominantly in postmenopausal women. Sarcoidosis and amyloidosis are other chronic inflaurmatory diseases that cause xerostomia. Other systemic diseases that can cause xerostomia include rheumatoid arthritis, sl stemic lupus erythematosus, and scleroderma. Remember: Xerostomia is the most common toxicity associated with radiation therapy to the head and neck. Comrnercially available saliva substitutes in general, contain an agent to lncrease vlscos- iq. such as carboxymethylcellulose or hydroxymethylcellulose, minerals such as calcium and phosphate ions and fluoride, preservatives such as methyl- or propylparaben, and fla- \ oring agents. Examples include: Xero-Lube,i Salivarti and Optimoist.s

A patient presents to your clinic with multiple exophytic masses covering the buccal mucosa, tongue and lips. A biopsy reveals that these are mucosrl neuromas. The most importrnt rerson this patient should be referred to a physician is bectuse of the risk of related: Squamous cell carcinoma of the tongue Pituitary hyperplasia Medullary carcinoma of the thyroid Sipple's syndrome

Medullary carcinoma of the thyroid The multipfe endocrin€ neoplasia syndromes (olso called MEN Syndrome./ have been classified into three distinct syndromes, each is inherited as an autosomal dominant trait: . \ten I: consists of tumors or hyperplasia of the pituitary parathyroids, adrenal cortex, and ofthe pancreatic islets. c'alled Sipple's Syndrome ard sabn?e lr: is characterized by parathyroid hyperplasia or adenoma, but no tumors to the pancreas. However, in addition, these . \len Il (also patients have pheochromocltomas ofthe adrenal medulla and medullary carcinoma of the thyroid gland. . \Ien III (elso kno$n os sub\tpe IIB): rs characterrzed by mucocutaneous neuro- mas, pheochron'rocytomas ofthe adrenal medulla and medullary carcinoma ofthe thyroid gland. Important: MEN I and II are related to MEN lll in that patients with types I and II syndromes have neoplasms ofvarious endocrine organs, but do not have the oral manifesta- rions ofmucosal neuromas. These oral lesions are most common on the lips, tongue, and buccal mucosa. \ote: The most important aspect of this syndrome is the medullary carcinoma of the thvroid because of its ability to metastasize and cause death. Therefore, the detection of the rnucosal neuromas may alert the clinician for early diagnosis and treatment.

A l0-year-old boy comes with ber mother to the dental office because of a painless swelling ofhis maxilla. Radiographic exam reveals an irregularly shaped radiopoque mass with a ground glass appearance. No other bulges have been noted by the mother. A biopsy reveals fibrous tissue in the bone. What is the most likely diagnosis? Monostotic fibrous dysplasia Polyostotic fibrous dysplasia Albright's syndrome Jaffe syndrome

Monostotic fibrous dysplasia Fibrous dysplasia is an idiopathic condition in which normal medulJary bone is gradually rcplaccd by an abnormal fiborus connective tissue prolifcration. The mesenchymal tissuc contains \ arllng amounts ofosleoid and osseous mateial that presumably arises through metaplasia. Thc discase most commonly presents as an asymptomatic. slo$' enlargcmcnt of the involved bone. I! may . jnvolve one bone or scvcral bones concomitantly. \Ionostotic fibrous dysplasia: is the most common, comprising 707o of cases. There is an equal male to female ratio, and this form is the most likcly to quiesce at puberty. A typical monostotic les- ion Nill involve the fcmur, tibia or ribs, with 2570 occu.ring in the bones ofthe skull flle mdiil/a r.r .atnnnlv involved). A panorex will show a radiopaque mass with inegular bordcrs that has a glass" appearance. See picture #,19 in booklet . Poltostotic fibrous dysplasia applies to cases in which more than one bone exhibits evidence ofthe disLrrder. It is rclatively uncommon; horlcver, many patients have lesions of the skull, facial boncs, or ii\\ s. as a component ofthe condition. Note: Alkaline phosphatase may be elevated in up to 307o oipalicnts $ ith polyostotic fibrous dysplasia, and a dramatic risc may herald malignant degeneration. Cal.rum and phosphorus tcnd to be normal. Important: Fibrous dysplasia has a variable radiographic appearance that ranges from a radiolucent le- sron to a densely radiopaquc mass. The classic ptesentation has been describcd as radiopaque with nu- merc'us bony nabeculae imparting a "ground-glasss" jaws. appearance. An important distinguishing feature L)ifibrous dysplasia is the poorly delined radiographic and clinical margins ofthe lesion The process iecms lo blend into the surrounding normal bone without cvidence of a circumscribcd border. \lalignant degeneration occurs in less than l7o ofcases offibrous dysplasia. Malignancies are almost exciusi\,ely osleosarcoma. For unknown reasons, monostotic and craniofacial Iesions have the greatest po- tential formalignant dcgeneration, and radiation therapy has been found to incrcase the risk by 400-fold. \ote: The differential diagnosis of fibrous dysplasia of the "ground jaws includes the ossilying fibroma, how- eyer, radiographically thc ossifying fibroma has a well-circumscribed appearsnce. Remember: N{cCunc-Albright syndrome is a designation that has bccn applied to patients with polyos- totic fibrous dysplasia, cutaneous melanotic pigmcntation rnaTlties (rnost cotwtronly precocious sexual devlopuent in.lbmales). fcate .r lait macules), and endocrine abnor-

All of the following are clinical features of the ossifying fibromas EXCEPT one. Which one is the EXCEPTION Slow growing expansile lesion More often in maxilla Asymptomatic Common in young adults around 35 years ofage

More often in maxilla This is false; these lesions, with rarc cxccptions, arise in tooth-bearing regions ofthc iau s. most ofien in the mandibular premolar-molar area. See picture #89 in booklet Thc ossifling libroma is a bcnign fibro-osseous lesion of the jaws that is considercd by nran\ in\estigators to dcvclop fiom undiffercntiated cells ofthe periodontal ligament. The :imilaritics between this lesion and thc ccmcntifying fibroma are numerous. Both tumors occur in similar age groups and locations and manifest comparablc clinical characteristics. The ossif-ing fibroma is a slow-growing, expansile lesion that is usually asymptomatic when d:scorered. Thcsc lcsions, with rare exceptions, arise in tooth-bcaring regions ofthejaws, most oflen in the mandibular premolar-molar area. They are uncommon lesions that tend ro occur in fcmales during the third and fourth decades of life. The most important radiographic feature ofthis lesion is the well-circumscribed radi- rrluccncy \r'ith a sharply defined border Thcy prcscnt a variable appearance dcpcnding on thc maturation or thc amount olcalcification present. As thc lcsion matures, it eventually becomes a relatively uniform radiopaque mass. The tumor consists ofa collagenous stroma that contains varying numbers ofunifbrm spindled or stellate cells. Irregular trabeculae of woven immature bone are most consistently rored in th€se tumors. The treatment ofossifying fibroma is most often accomplished by sur- gical removal utilizing curettage or enucleation. Recurrence is uncommon. .\ r ariant of ossifoing fibroma, thejuvenile ossifying fibroma, has been described in children and teens. This rare lesion behaves in a more aggressive fashion than does the ossifliing fi- broma, and it may require more extensive therapy when encountered.

All of the following may be seen in a patient with cerebral palsy EXCEPT Difficulty with mastication and swallowing Higher incidence of periodontal disease and caries Attrition of the teeth Multilocular radiolucencies of the jaws

Multilocular radiolucencies of the jaws Cerebral palsy is a term used to describe a group ofdisorders affecting body movement and muscle coordination. It is due to an insult to or anomaly ofthe brain's motor control centers. This damage interferes with messages from the brain to the body, and from the bod\ to the brain. The effects vary widely from individual to individual. Cerebral palsy rs characterized primarily by spastic paralysis or impaiment of control or coordination !r\ er \ oluntary muscles and is often accorrpanied by mental retardation, seizures and dis!rrders of Yision and communication. \ote: No intraoral anomalies are unique to persons with cerebral palsy. However, several conditions /r/rose /isted on the the normal population. ltont ofthe card) are more comnton or more severe than in Remember: Down syndrome is a congenital defect caused by a chromosomal abnor- rnaliy.- (trisomy 2 l). It is marked by various degrees of mental retardation and character- istic phvsical features such as a short, flattened skull, slanting eyes, a thickened tongue r.tissured), broad hands and feet and other anomalies. Other oral rnanifestations include mandibular prognathism. increased incidence ofperiodontal disease, delayed eruption of teeth. higher incidence of congenitally missing teeth, malocclusion, and enamel dyspla- s ia.

A 62-year-old patient comes into your oflice complaining of his loose lower molars. The health history reveals recent lower back pain. On a hunch, you send him down the hall to the orthodontist to take a lateral skull radiograph which reveals "punched-out" radiolucencies. You will refer this patient to the physician suspecting a diagnosis of: Non-Hodgkin's lymphoma Hodgkin's lymphoma Multiple Myeloma Langerhan's Cell Disease

Multiple Myeloma Iultipfe mleloma lso known as "Plastna Cell Mveloma') is a disease ofabnormalplasma cells that mosr often build up in the bone marrorv. The cells form tumors in many bones ofthe body. Clinical Features: . \Ien 2:l (40-70 f.r even if the skin rears o/r// Vcrtcbrae, ribs, and skull are most frcquently involvcd; pain in lumbar or thoracic region is a common early symptom. . Jaws are rarely a primary site, but become involvcd in 70% of the cases, molar ramus area most common site. Symptoms include swelling, pain, loosening ofthe teeth, and paresthesia. Radiographic features: variable; slight demineralization to extensive dcstruction, characteristic finding rs multiplc. small, discreet "punched out" radiolucencies in involvcd bones ln a patient :u:pccted ofhaving multiple myeloma. a lateral skull radiograph is best to confirm the diagnosis. Laborator-'- fi ndings: important in establishing diagnosis: . Abnormal monoclonal immunoglobulin protein peak known as an M spike. The immunoglobulin is usualll ofthe IgG or IgA class. with a monoclonal Jight chain cofrponent. . . Lrinary monoclonal light chains, so-called Bence Jones proteinu a Anemia Treatment and Prognosis: chemotherapy, radiation; prognosis poor. Important: A form ofamyloidosis occurs in l0% of multiple myeloma patients. It is due to the forma- rron of complex proteins in which immunoglobulin light chains are precursors. Amyloid ptotein is de- posited in various organs an can lead to organ dysfunction f€speciallr kidnq', hedfi, GI ltucl, liver and I Plasmacytoma is a localized collection ofmonoclonal plasma cells. The discasc is dividcd \61*,-'intoprimaryplasmacytomaoftheboneandextramedullaryplasmacytoma.Thcimportance - ,.:rr' i.';$t:l: ofthe diagnosis restswith thc potential forthese disorders to progress to multiple myeJoma. 2 Primary presentation of intraoral non-Hodgkin's lymphoma is uncommon. In gencral, the oral manifestations occur secondary to a morc widespread distribution throughout the bodY 3. Hodgkin's disease involving the oral cavity is also considered a raity, but it still can ap- pear here initially.

A 25-year-old college student comes into your office cornplaining that when she wakes up she has trouble opening her mouth. When conducting a TMJ exam, you note tenderness ofthe right lateral pterygoid and non-reciprocal clicking of the right TMJ. What is the most likely cause of the patient's myofacial pain? Trauma Muscle spasm Periodontal disease Tumor

Muscle spasm Such spasm may arise in one ofthree ways: muscular overextension, muscular over- con- traction or muscle fatigue. The most frequent cause ofthe spasm seems to be muscle fa- tigue. This syndrome is seen predominantly in women, usually in the 20 to 40 age range, and generally occurs unilaterally. There are four cardinal signs and symptoms ofthe syndrome: 1. Pain 2. Muscle tendemess 3. A clicking or popping noise in the TMJ 4. Limitation in jaw motion (especially in the morning) The pain itself is usually unilateral and is described as a dull ache in the ear or preauricular area, rvhich may radiate to the angle ofthe mandible, temporal area or lateral cervi- cal area. The muscle most apt to exhibit tenderness is the lateral pterygoid muscle. \ote: There are no radiographic findings associated with MPS (mtofacial pain syn- droDte) Treatment: Most cases are selflimiting. Soft diet. limited talking, no gum chewing, moist heat. NSAIDs and Diazepam help relieve symptoms.

Which of the following is a relatively rare autoimmune disorder of peripheral nerves in which antibodies form against acetylcholine (ACh) nicotinic postsynaptic receptors at the myoneural junction Myasthenia gravis Myelofibrosis Multiple sclerosis Graves' disease

Myasthenia gravis Myasthenia gravis is an autoimmune disorder in which antibodies form against acetylcholine (ACh) nicotinic postsynaptic receptors at the myoneural junction. The muscles are quickly iatigued with repetitive use. lt is tlpical for a myasthenic patient to have a flat- tened srrile and droopy eyes. with slow papillary light responses. Xerostomia and ram- pant caries may accompany myasthenia gravis. The acetylcholine that is necessary for the proper transmission ofnerve impulse is destroyed, with the result that salivary glands do not receive adequate stimulation. Note: Head and neck manifestations include inability to focus eyes, drooping eyelids, double vision, difficulty in chewing and swallowing, and slurring ofwords. Multiple sclerosis is a chronic, often disabling disease that randomly attacks the CNS lhrain ancl spinal cord). It is believed to be due to an autoimmune response in which the inxnune system attacks a person's own tissue. Twice as many women as men have MS, s ith the onset of symptoms occuring most often between the ages of 20 and 40. Symp- toms may range from tingling and numbness to paralysis and blindness. Patients with multiple sclerosis sometimes have facial andjaw weakness. In addition, both Bell's palsy and trigeminal neuralgia may develop more frequently in patients with MS.

Which of the following are typically within soft tissue and may mimic inflammatory lesions of odontogenic origin? Median alveolar cyst Globulomaxillary cyst Nasolabial cyst Nasopalatine cyst

Nasolabial cyst The nasolabial cyst has been called, inaccurately, nasoalveolar cyst. This latter designa- tion is inappropriate because the entity is not a true cyst ofthe maxilla. Rather, it repre- sents a soft tissue cyst without involvement ofthe alveolus, hence the prelerence for the designation "nasolabial cyst." Note: Because thjs cyst is extraosseous, it is not likely to be seen on a radiograph.

The circular radiolucent area seen in this radiograph is clinically seen as a marked swelling in the region of the palatine papilla. It is situated mesial to the roots of the central incisors. The pulps of the anterior teeth in this patient tested vital. These findings would be compatible with what diagnosis? SEE PIC ON PAGE 89

Nasopalatine duct (canal) cyst Of the cysts ofthejaw, those that arise from epithelial remnants in the incisive canal are the most common type of maxillary developmental cyst. Histologically, this cyst is lined rvith vessels, nerves and mucous glands in the wall. They most often remain limited as to size and are asymptomatic. Some of them, however, become infected or show a tendency to grow extensively. When tl'tis occurs, surgical intervention is indicated. less common) v^ri^nt of the nasopalatine canal cyst is the cr st of the palatine papilla. \ote: The soft tissue (ancl.far \\'hen making a diagnosis ofthis cyst, the following two cysts should be ruled out: . The globulomaxillary cyst: usually appears between the roots of the lateral inci- sor and those ofthe canine. lt is "pear-shaped" reerh to diverge. See picture f87 in booklet . and often causes the roots ofinvolved The median palatal cyst: usually situated in the midline of the hard palate, poster- ior to the premaxilla. Clinically, Ihis lesion presents as a firm swelling, which is usu- ally painless. Note: Some investigators now believe that this cyst represents a more posterior presentation ofa nasopalatine duct cyst, rather than a separate cystic degeneration ofepithelial rests at the line ollusion of the palatine shelves. See picture ff48 in booklet

In which case is an incisional biopsy indicated? 3 mm well-encapsulated fibroma Necrotizing sialometaplasia ofthe hard palate 2 mm papilloma of left commissure of lips Aneurysmal bone cyst

Necrotizing sialometaplasia ofthe hard palate A biopsy is a procedure perfonred to remove tissue or cells from the body for examina- tion under a microscope. Biopsies are usually performed to detemine whether a tumor is malignant or to detemine the cause of an unexplained infection or inflammation. When the entire tumor is removed, it is called excisional biopsy technique. lfonly a portion ofthe tumor is removed, it is called incisional biopsy technique. An incisionaf biopsy (also called diagnostic biop.st) is done when lesions are too large to excise initially without having established a diagnosis or are of such a nature that ex- cision would be inadvisable. An excisional biopsy is preferred ifthe size ofthe lesion is such that it may be removed along u ith a margin ofnomral tissue and the wound can be closed primarily. Erample: A l -cm ex- ophl-tic mass flfrich is a lesion that grovs ouf,rordfrom an epithelial surface) on the cheek. l. Biopsy is the most reliable technique to diagnose soft tissue lesions. \ote* 2. The fixative ofchoice to preserve biopsy specimens lor routine histological examination is l0%o buffered formalin. 3. The scalpel is the instrument of choice since it cleanly removes the tissue and does not dehydrate it as cautery or the high-frequency cutting knife may. .1. The rationale for surgical removal and biopsy of a large periapical lesion suspected to be of inflammatory origin is that a clinical diagnosis can be con- firmed microscopically. This is the only way to distinguish between a granuloma and a cvst.

Which of the following is probably the most Traumatic neuroma Neurilemmoma Neurofibroma Nodular fasciitis Fibromatosis

Neurofibroma Neurofibromas may appear as solitary Iesions or as muitiple lesions as part ofthc syndrome neurofi- bromatosis 6,()// ne. klinghdusen:\ disease of skin). lr rs a benign ncoplasm, howcver the etiology of solitary neurofibromas is unknown. Most researchers believe the cell oforigin is the Schu'ann cell; oth- ers believe dre pe neLlral fibroblasr is rcsponsiblc- For the solitary neurofibroma thc tongue. buccal mu- cosa, and vestibule are the oral regions most commonly affected.

Patient goes to dermatologist with light brown birthmark like spots on his back. A thorough scan reveals freckles on his armpits and pigmented lesions within the iris. Intraorally patient has multiple lesions on gingiva and alveolus. Whats the most likely diagnosis Neurolemoma Neurofibroma Neuroma Fibroma

Neurofibroma \bn Reckfinghausen's disease (neut'oJibromatosis) is an autosomal dominant disease associated rvith loss ofa tumor suppressor gene (NF/ or NF2l. lt is characterized by multiple neurofibromas of the oral cavity and on the skin, and pigmentations of the skin r Ccrfi-au-luit .rpor.s/, iris freckling (Lisch spots), and axillary freckling Crou'e )s sign.). \ote: The presence ofsix or more caf6 au lait macules greater than 1.5 cm in diameter is generally regarded as being indicative of this disease until proven otherwise Radiographic features: . \\ ell-demarcated unilocular or multilocular radiolucency . . Possible root disturbance Possible Treatment: . . jarv enlargement Surgical excision but may be impractical due to the number of lesions Best 1eft untreated because multiple recuffences may be associated with malignant transformation to neurogenic sarcoma to l5%o.1 of mtlignanl transformation \ote: The importance of the lesions is the high risk (5%' Remember: The single neurofibroma presents at any age, it comnronly appears as a ses- sile, firm, pink nodule that commonly occurs on the tongue, buccal mucosa and lestibule. It is rerloved by surgical excision and rarely recur.

Type I dentinogenesis imperfecta is associated with which genetic disease OI Marfar's syndrome Ehlers Danlos Cystic Fibrosis

OI The characteristic features olosteogenesis imperfecta (OI) vary greatly from person to person and not all characteristics are evident in each case. The chief clinical character- istic ofosteogenesis imperfecta is the extreme fragility and porosity ofthe bones, with a proneness to fracture. Other features include: pale blue sclera, deafness due to otosclerosis, abnormal teeth, loosejoints and low muscle tone. a triangular face, and a tendency toward spinal curyature. Clinically the teeth have: . . . Crowns that are bulbous: with a cervical constriction Pulps that are obliterated, either partially or completely Roots that are narrower and shorter *** The deciduous teeth are more s€ver€lv affected that the permanent T! pes of ost€ogenesis imperfecta: . . . . Type l: most common and mildest fonn Type ll: most severe lonn resulting in multiple fractures Type lll: most severe form beyond the perinatal period Type IV: mild to moderately severe bone fragility just dentition from birth process There is no known cure for osteogenesis imperfecta. Treatment is directed toward pre\ enting or controlling the symptoms. \ote: The cause of osteogenesis imperfecta is believed to be due to a genetic defect that causes imperfectly-formed, or an inadequate amount ofcollagen. The affected person has either less collagen than normal, or a poorer quality of collagen than normal, leading to \r eak bones that fracture easilY.

A new patient walks into your office. Your initial physical assessment reveals that his eyes are set wide and that he has multiple lesions of the skin. When shaking his hand, you notice that the skin of his palm is very thick and has palmer pitting. When doing a health history, he reveals that he sees a neurologist and that he has some calcified structures "in his brain". A panoramic radiograph may reveal Osteomas Odontogenic keratocysts Odontomas Dentigerous cysts

Odontogenic keratocysts This patient has nevoid basal cell carcinoma syndrome, which is an autosomal dominant disorder characterized by oral, systemic and skeletal anomalies. It is an inherited group ofdefects which involve abnormalities ofthe skin, eyes, nelvous system, endocrine, glands and bones. The condition is characterized by an unusual facial appearance and a predisposition for skin cancer. It is also krown as the basal cell nevus-bifed rib syndrome, the basal cell nevus syndrome and the Gorlin and Goltz syndrome. Note: Recently, mu- tations in the human homologue ofDrosophila patched were identified as the underlying genetic event in this syndrome. Possible abnormalities include: . fPICIl), a tumor suppressor gene Cutaneous anomalies: including multiple basal cell carcinomas, other benign dermal cysts and tumors, palmer pitting, palmer and planlar keratosis and dennal calcino- sis. . D€ntaf and osseous anomalies: including odontogenic keratocysts (o.ften multiple) mild n.randibular prognathism, rib anomalies (often bifid), and venebral anomalies. . Ophrhalmologic abnormalities: including hypertelorism with wide nasal bridge an congenital blindness. . Neurologic anomalies: including mental retardation, dural calcification, agenesis of corpus callosum and congenital hy&ocephalus. . Sexual abnormalities: including hypogonadism in males and ovarian tumors in fe- males. Radiographic features: . . Calcification ofthe falx cerebri Presence of odontogenic keratocysts

The congenital absence of many, but not all, teeth is known as: Hypodontia Oligodontia Diphyodontia Anodontia

Oligodontia Tlr o forms of anodontia: L Complete true: is a rare condition in which alt of the teeth are missing. It may in- volve both the primary and permanent dentitions. It is usually associated with heredi- tarv ectodermal dysplasia. See picture #l in booklet :. Partial anodontia (commonly referred to as congenitally missing teeth): is rather cor non. Teeth usually affected include the third molars (maxillary nandibular), maxillary lateral incisors ald mandibular second premolars. Note: As a general rule, if only one or a few teeth are missing, the absent tooth will be the most distal tooth ofany glen type (if molat then it would be the third molar). See picture #2 in booklet Other terms to b€ familiar with include: . Ofigodontia: refers to the congenital absence of many (usually six or more) but not all, teeth . . Hypodontia: refers to the absence ofonly a few teeth Diphyodontia: is having 2 successive sets of teeth (as in humans) as opposed to pol)?hyodontia fraving . more than two sets of teeth in a ldetime) Hypsodontia: having teeth with high crowns iated with a diet ofabrasive foo<is

Mandibular tori most often appear In the retromolar pad region On the lingual surface of the mandible, most often in the premolar region On the lingual surface of the mandible, inferior to the mylohyoid ridge Along the midline of the hard palate

On the lingual surface of the mandible, most often in the premolar region Tori and exostoses are nodular protuberances of mature bone whose precise designation depends on anatomic location. These lesions are of little clinical significance; they are non-neoplastic and rarely are a source ofdiscomfon. The precise etiology ofthese lesions remains obscure, although evidence has been presented to suggest that the torus may be an inherited condition. Note: The etiology ofexostoses is also unklown. It has been suggested that the bony growths represent a reaction to increased or abnormal occlusal stresses ofthe teeth (i.e., bruxism) in the involved areas. Mandibular tori (also called torus mandibularis) are bony. exophytic growths that occur along the lingual surface ofthe mandible superior to the mylohyoid ridge. Vandibular tori may occur singly, however, there is a marked tendency toward trilateral occurrence, and the lesion is not necessarily confined to the premolar region. Unlike palatal tori. the mandibular tori are more readily demonstrated radiographically. forr huoxillary and mandibulor) are ofno pathological significance and rarely are they erf clinical significance r.vhile the normal teeth are still present. If, howeyer, a complete denture needs to be made, they should be carefully removed.

The ? presents as a movable painless submucosal nodule with a yellowish pink discoloration most commonly in the floor of the mouth. Lipoma Ranula Lymphoma Oral lymphoepithelial cyst

Oral lymphoepithelial cyst Almost all oral lymphoepithelial cysts arc lcss than 0.6 cm. in diameter at thc timc ofdiagnosis, which is usually during the teen years or the third decade oflife. The site most colnmonly affected is thc floor of the mouth, but the lateral and ventral tongue are not uncommon sites ofoccurrcnce. nor is thc soft palarc, espccially lhe mucosa abovc thc pharyngcal tonsil. This cyst has a clinical appearance similar to that ofan epidermoid cyst or a dcmroid cyst ofthc oral/phanngcal mucosa, but its growth potential is much lcss than ihc othcr cysts. Lymphomas arising rvithin the oral cavity account for less than 59/o oforal malignancies. In the head and ncck. most lymphomas start in the rcgional Iymph nodes or rvithin cxtranodal lymphoid sites in areas known as mucosa-associated lymphoid tissues (MALT h'nphonrur./. Within thc oral cavity. lymphoid tissue is chiefly represented in Waldeyer's ring fu ring oflvnphdtic tissue fonied br lhe tvo palatine tonsils. Ihe phanngeal tonsil,theli gual tonsil, dnd inteh^ening htnphoid tissre). Ofimponancc rclative to lymphoma is that in both nodal and extranodal sitcs, neoplasia can occur A ftrndamental aim is Io separatc lymphomas into two grollps: . Ilodgkin's t)pe: is charactcrizcd by painless enlargcmcnt of lymph nodes or extranodal tissue. \\'ithin Ihc oral cavity, tonsillar enlargement. usually unilateral. may be seen in the carly phascs. His- tologicalll-, cominon to all forms ofHodgkin's diseasc is the prcscncc ofmalignant l)'rnphoid cells and non-ncoplastic inflamrnatory cells. including Iymphocytes, macrophages, eosinophils. and plasma cclls. Ofgreatest significance is thc identification ofthe Reed-Sternberg cell, which must be present fbr thc diagnosis of Hodgkin's disease to be established. Note: The Ann Arbor system of clini- cal stsging is uscd to determine the clinical extent ofthe disease and is an impofiant factor fbr dcciding !he tJ-pe and intensity oftherapy and helps dcterminc thc prognosis. . \on-Ilodgkins tlpei when primary oral soft tissues lesions are present (\'hich i.t rare). thcy are charactcnzed by an absence ofsymptoms and by a relatively soft charactcr, oftcn $'ilh overlying r.rl- .erations. If bone is the primary site. alveolar bone loss and tooth mobility are oftcn prcscnting .rgns. S\!elling. pain, numbncss ofthc lip. and pathologic fracture may also be associatcd with bone lesrons. In general. thcrc arc t\r'o groups ofnon-Hodgkin's lymphona noddar (bllicular"/ and diffuse ibrms. For non-Hodgkin's Iymphomas. numerous classification schemes have cvolved. Thcsc in- ;lude the classifications ofLuke and Collins, the WHO, Rappaport, and thc NIH International Work- lns Fonnulation.

A fifteen-year patient presents to his physician because of localized pain in his right femur and rapidly enlarging swelling. A radiograph ofthe area shows a "sun-ray" appearance, Which is a likely diagnosis for this patient based on incidence? Chondrosarcoma Osteosarcoma Sclerodema Chronic osteomyelitis

Osteosarcoma Osteosarcomas bl.so called osteogenic sdrcomas) account for approximatcly 20Yo of all sarco- mas and, alier plasma ccll rnycloma, arc thc most common primary bone trnnors. Conventional ostcosarcomas involving thc rnandible and maxilla display a predileclion for malcs arising in the jaws have a mean age of34 years. The majority ofnrandibular osteosarcomas arise in thc body ofthe mandible. In the maxilla, there is nearly equal incidcnce ol-tumors involv- ing the laveolar ridge and maxillary antrum, u,ith feu, a1'ficcting the palate. (60o/o) Osteosarcomas involving the mandible prcscnt most commonly with swelling and localized pain. ln some cases, there may bc loosening and displacement of teeth as well as paresthesia duc to involvement of the inl'crior alveolar nerve. Maxillary tumors display similar clinical symptoms but nlay cause paresthesia ofthe infraorbital nervc, epistaxis. nasal obstnlction, or cyc problems. Radiographicalln early ostcosarcomas nray be characterized by localized widening ofthe peri- odontal ligament space of onc or two tccth. Advanced tumors can be visualized as moth-eaten radiolucencies or irrcgular. poorly marginated radiopacities. See picture #42 in booklet Osteosarcomas are best treated by radical mandibulectomy or maxillectomy, with radiothcrapy and chemotherapy for rccuncnces, soft tissue extension, or metastatic diseasc. As with most maLgnanl jaw tumors. initial radical surgery resulls in superior survival ratc of 807o compared with l'o D sunival lbr local surgery. Osteosarcomas ofthc jaws frcqucntly recur n'relastalic rate of 25 to 50o/o. The most common sites ofmetastases arc lung and brain. \ote: Prognosis is better for mandibular tumors than lor maxillary tunors. l/.;/0 \ote: Chondrosarcomas arising in the mandible and maxilla are extren'tely mrc and havc countcd li'r ipproximately l9lo of all chondrosarcomas ofthe entire body. Cartilagc is produced by tumor .ells. The most common signs are a painless swelling and expansion ofthe affected bones, re- :rltrng in loosening ofteeth or ill-fitting dentures. Radiographically, the lesions vary fiom mothlaten radiolucencies that arc solitary or multilocular to diflusely opaquc lcsions. The trealment is ir rdc Iocal or radical excision. Note: They are radioresistant.

An old patient presents to your oflice with ill-fitting dentures. Radiographs reveal hypercementosis on roots and the patient is speaking embarrassingly loudly to you that she is more conscious ofhis baldness now that his hat doesn't fit anymore." These signs all point to what possible diagnosis? Paget's disease Osteosarcoma Fibrous dysplasia Albers-Schoenberg

Paget's disease Paget's discase ofbonc is a chronic, slouly progressive condition ofunknown ctiology(it appears toie lanitiatl. The most frecluent sitcs ofinvolvemenl include thc spine' l'elnur' cranium' pelvis' and stemum. Tbe bone may become dcnse, but liagilc, because of cxcessive breakdou'n and fbr- mation ofbonc. The disease is morc conmon in males and affccts adults ovcr thc age of 50. Thc signs and symptoms inclucle pain in thc aflectcd arca, defomlily of tlre bone in thc irffected area' su"sceptibility to fractures in lire affected arca' and headache and hearing loss ifthe affccted arca is the skull. Note: Thesc symptoms dcvelop slowly' When the . jarvs are affected: Patient who rvears a maxillary denture cotnplains that thcy do not fit aS thc n-Iaxilla progres- sively cnlarges . . . Paqet's tlisease ofbone is also called Osteitis Deformans' See picture #32 in booklet Ultimatelyl the alveolar ridge wiclens. r,irh a rclative flatrening ofthe palatal vault When tecth are prescnt, incrcased spacing as wcll as loosening is notcd Hypcrcemcntosis of tooth roots, loss of lamina dura, oblilcralion of the PDL space and resorption ofthe roots ]licroscopically, ostcoblasts and multinuclcated osleoclasts arc lbund in abundance As the lcsion adr ances. dense bone wlth numeious reversal or growth lines are sccn' giving the tissue a mosaic parrem. Other important features of this disease include: . Palicnts may also give a history ofprogrcssively increasing sizc ofhats or nc'$ dentttrcs being made at progiessively more frequent intervals. Note: This is due to bony changes' . Boncs are warm to touch due to incrcased vascularity .\-raysofthesku|landthejawsdcmonstlatethctypical''cotton-wooll.appearance . Lab iests: Drastically increased serum alkaline phosphatase. Scrum phosphale and calcium are normal. Urinary calciun and hydroxyproline are increased' as parathofmone antagonlsls ha\ e bcen eilective in suppressing bone resorption and dcposition lt is seldom latal

Where in the oral cavity is malignant melanoma most commonly found Tongue and mandibular alveolar ridge Buccal mucosa and pharyngeal pillars Palate and maxillary gingiva Mucobuccal fold of lower lip

Palate and maxillary gingiva lalignant melanoma accounts for only 4% of all skin cancers; however, it causes the createst numbcr ofskin cancer rclatcd dcaths worldwide. Clinical features: . Malc' morc oftcn than fcmalcs . . L sually palalc or rnaxillary gingiva 5th decade or older . A. B. C. D's of melanoma: -\ = asymmctry B = border inegularity C : color variability (brown, black, blue, gray, pink) D = diameter of greater than l/4 inch Excessive exposure to UV radiation from the sun may bc the primary cause ofmclanoma. /o/ief \falignant melanoma has bcen linked to both a lot of sun exposure over a lifetime and to painful sunbums during childhood. \{alignant mclanoma is an uncommon neoplasm ofthe oral mucosa. It exhibits a definite predilcction for the palat€ and the maxillary gingiva/alveolar ridge. Unfortunately, oral mucosal melanomas have a dismal prognosis. See picture #36 in booklet \ote: A nevus is a mole. Almost all moles are normal. Atypical (dl,splastic) nevi are unusual moles that are gencrally larger than normal moles and are eithcr flat or have a flat part. They have irregular bordcrs and oftcn are variable shades olcolor, particularly brown. The presence ofdysplastic nevi may mark a greater risk ofmalignant mclanoma developing on apparently normal skin. See picture #46 in booklet

A 73-year-old patient presents to your office with complaint of a lasting ulceration on the right side of his tongue. He has a history of hypertension and high cholesterol and a 40-year pack history of srnoking. The patient takes anti-hypertensives and anti-lipidemics and has no allergies. You cannot find any sources of trauma. After two weeks, the ulcer has grown in size. What is the likely diagnosis of this most common malignancy of the oral cavity? Adenoid cystic carcinoma Mucoepidermoid carcinoma Basal cell carcinoma Squamous cell carcinoma

Squamous cell carcinoma x Squamous cell carcinoma is a malignant neoplasm ofstratified squamous epithelium that is ca- pable oflocally destructive growth and distant metastasis. It is the most common type oforal cancer, accounting for over 90%o ofall malignant neoplasms ofthe oral cavity. lt is two times more prevalent in m ales Possible sites: . . (40-65 years of4ge./. See pictures #44 and #45 in booklet. Lower lip (most common site) TottgU.e (posterior leteral border is the most common loccttion, dorsum least common lo- cqtiotr) . . . . Floor ofthe mouth (least Sofl palate (u conmon) Gingiva/alveolar ridge Buccal mucosa Clinical f€atures: . . . . favorqble plognosis) Early presentation ofleukoplakias and erythroplakias Painless ulcer, tumorous mass, or verrucous (papillary gro\tth) Occasional loosening or loss ofteeth Possible paresthesia ofthe teeth and lower lip Risk f'actors identified include smoking, alcohol consumption, painful and ill-fitting dentures, chronic inflammation and the use ofsmokeless tobacco. Important: Tobacco use is the pri- mary risk factor Note: Causes ofgenetic alterations which results in loss ofcell cycle con- trol. abnonnal signaling, increased cell survival, and cell motility include tobacco, human papillomavirus (subq,pes 16 and l8), and heredity. Remember: The most reliable histologic criterion for a diagnosis oforal squamous cell car- cinoma is invasion,

Of the following types of squamous cell carcinoma which is the least common Squamous cell carcinoma of the palate Squamous cell carcinoma of the nasopharynx Squamous cell carcinoma of the oropharynx Squamous cell carcinoma of the maxillary sinus

Squamous cell carcinoma of the nasopharynx

A 48 year old female patient walks into your office. She states that she is diagnosed with some disease which she can't remember the name of. Her physician wants her to follow up with you, her dentist, regularly to watch out for cancer of the tongue and throat. She also has a bald tongue, and states that her fingernails "look funny." What disease does she have? Aplastic anemia Plummer-Vinson syndrome Pernicious anemia Cushing's syndrome

Plummer-Vinson syndrome Plummer-Vinson syndrome characterized by an iron-deficiency anemia, atrophic changes in the buccal, glossopharyngeal, and esophageal mucous metnbranes, koilonycha (spoon-shaped./inger rail.s), and dysphagia. The dysphagia is due to an esophageal stric- ture or web. SCC of the tongue and throat are complications. lt is most common in mid- dle aged women, rarely in the male. The etiology unknown. Because ofthe predisposition to the development ofcarcinoma ofthe oral mucous membranes, it is essential that the di- agnosis be established early so that treatment can be given ASAP This includes administration of iron, vitamin B complex and a high protein diet. Aplastic anemia is a form of anemia in which the capacity of the bone marrow to gen- erate red blood cells is defective. Two types: l. Primary: unknown cause, affects young adults. The signs and symptoms include pallor, weakness, malaise, dyspnea (dilfcul4, breathing), headache and vertigo. Oral symptoms include spontaneous bleeding,bnstsing (petechlaeT and gingival infections. It is usually fatal. 2. Secondary: caused by exposure to toxic agents, such as radiation, chemicals or drugs (fbr exanple, chloramphenicol). It can occur at any age. Symptoms are the same as primary. Prognosis is good once you remove the cause. *** Aplastic anemia is the most serious and life-threatening blood dyscrasia associated s ith drus toxiciw.

The category of pigmented (usually purple) lesions in the skin caused by extravasation of blood from the capillaries is known as: Petechiae Ecchymosis Varicose veins Purpura

Purpura Purpura spots (pinpoint spots) are purplish discolorations in the skin produced by small bleeding vessels near the surlace ofthe skin. Purpura may also occur in the mucous mem- branes (such as the lining oJ the mouth) and in the intemal organs. Purpura by itselfis only a sign of other underlying causes of bleeding. Purpura may occur with either normal purpuras) or decreased platelet counts (thromboq)topenic purpltra.rr. Platelets help maintain the integrity of the capillary lining and are platefet counts hrcnlhrombocftopenic irnportant in the clotting process. Note: Large purpura spots (> 3 mn) are called ecchy- moses. Major kinds of purpura: . Thrombocytopenic purpura (Werlhol's tlisease): a bleeding disorder characterized by a deficiency in the number olplatelets. This resr.rlts in multiple bruises, petechiae, and hemonhage into the tissues . Thrombotic thrombocytopenic purpura (TTP): a severe and frequently fatal form characterized by a low platelet count in the blood due to consumption of platelets by thrombosis in the terminal arterioles and capillaries ofmany organs Oral manifestations of thrombocytopenic purpura: . Severe and profuse gingival hemorrhage . Petechiae occur commonly on the palate Important: Tooth extractions are contraindicated due to the tendency for excessive bleedin s.

You have a new patient in your dental oflice who has just from Denver. He says his doctor told him that he has some disease caused by living at a high altitude. When conducting an intraoral exam, you lind that his tongue is a deep purple and his gingiva bleed easily. What disease is a likely cause ofthese findings? Polycythemia Vera Hemophilia B Thallesemia Major Porphyria .

Polycythemia Vera Polycythemia is the condition oftoo many red blood cells in the circulation. The blood can be too thick to pass easily through the small blood vessels ofthe body. This in tum leads to clot formation and blockage ofthe small vessels which can lead to a stroke There are two types ofpolycythemia: I . Primary polycythemia (also called polycythemia vera or e4lhemia) occurs when excess erythrocytes are produced as a result ofturnorous abnormalities. This occurs in the tissues that produce blood cells. Usually accompanied by leukocytosis. Splenomegaly, as a result ofvascular congestion, is seen in 75% ofpatients. 2. Secondary polycythemia is an increase in the total number of erytluocytes due to another condition. For example. chronic tissue hypoxia of advanced pulmonary dis- ease, high altitude fo.r ler's disease) or the secretion oferythropoietins by certain tumors. Oral manifestations of polycythemia: . Oral mucous membranes red. . . (especialb tlrc gingiva and tongue) appear deep purplish- The gingiva are very swollen and bleed very easily. Submucosal petechiae and hematomas are common. (purplish spots), ecchymoses (same as petechiae, but bigger) \ote: Erl-thromelalgia is a rare syndrome of paroxysmal vasodilation with buming oain. incrcascd skin tcmpcrature, and redncss ofthc feet and, less often, thc hands.

The most common site of squamous cell carcinoma of the tongue is Dorsum Ventral surface Tip Posterior lateral border

Posterior lateral border Squamous cell carciloma i'SCC) accounts for greater than 9070 oforal cavity and oropharyngeal malignancics. Lip carcinomas account for 25,q/o b 30o% ofall oral cancers. Approximately 909/0 occur on the lower lip due to increascd sun exposure. IntraorallJ', the most common site is thc posterior lateral bor- der of the tongue, it is particularly uncommon to have lesions develop on thc dorsum or in the tip ofthe tonglle. Metastases from tongue cancer are relativcly common at thc time ofprimary trcatmcnt. In gcneral, metastatic deposits from SCC ofthe tongxe are found in the lymph nodcs ofthe neck. usually on the ipsilateral side. The first nodes to become involved are the submandibular orjugulodigastric nodes at the angle ofthe mandiblc. Noter The floor of the mouth is the second most common intraoral lesion of SCC.

Hypercementosis is the excessive deposition of secondary cementum on the roots. Any tooth may be involved however which teeth are most frequently involved? Molars lncisors Premolars Canines

Premolars Hypercementosis is often confined to the apical half of the root but, in some instances, may involve the entire root. In large majority of instances, it affecls vital teeth, is not as- sociated with any one particular systemic disease and may be regarded as a dental anom- aly. It rnay be seen when a tooth has lost its antagonist or when there is chronic inflammation of the tooth. The premolars are most frequently involved. Next in fre- quency are the first and second molars. Hypercementosis produces no significant clinical signs or symptoms indicative of its presence. It is seen radiographically as a bulbous enlargement that has surrounding it a continuous and unbroken periodontal membrane space and a normal lamina dura. See picture below. Note: There is a form of hlpercementosis which is a common feature in Paget's disease that involves thejaws. In addition to Paget's disease, hypercementosis is associated with supraeruption, apical periodontal infection, occlusal trauma, toxic thyroid goiter, acromegaly, and pituitary gigantism.

24 year old female patient comes in complaining of a 14mm exophtic red mass present on the gingiva between teeth 5 and 6. Lipoma Pyogenic granuloma Epulis granulomatosum Peripheral fibroma

Pyogenic granuloma L Thc peripheral fibroma is a rcactive hyperplaslic mass that occurs in the and may be dcrivcd from connective tissue ofthc submucosa or the PDL. It presents as well-demarcatcd lbcal mass \\ ith eithcr a sessile or pedunculated base. Ii is similar in Singiva qolor to the suffounding connective tissue. It may be ulccratcd. The treatment for a pcriphcral fibroma is local excision. Recurrence is rare' \ote: Other variant forms ofthe peripheral fibroma includc: . The peripheral odontogenic fibroma: which is gingival mass composed ofa well-vascularized, flbrous connective tissuc. Thc distinguishing featlre ofthis variant is thc presencc ofstrands ofon- dorogenic epithelium, oftcn abundant, throughout the connective tissue. It is usrlally non-ulcerated. . Thc peripher.l ossifying fibroma: is a gingival mass in which calcified islands. presumed to be bonc. are seen. The bone is found lasts. The surface is often ulccrated. See qithin a non-encapsulated prolifcration of plump benign fibrob- picture #85 in booklet :. Focal fibrous hyperplasia is hlperplasia oforal mucosa. It is also callcd traumatic fibroma, irri- larion fibroma. and hyperplastic scar. It is a rcactivc lcsion caused osually by chronic trauma to oral mucous membranes. The giant cell libroma is a focal fibrous hlT'erplasia in which connective tissue cells. many ofwhich are multinucleated, assume a stcllatc shapc

A child with achondroplastia has a Relatively normal torso and long arms and legs Short torso and long arms and legs Long torso and long arms with short legs Relatively normal torso and short arms and legs

Relatively normal torso and short arms and legs Achondroplasia is the most common type of dwarfism. The upper an.ns and thighs are more shortened than the forearms and lower legs. Generally, the head is large, the fore- head is prorrinent, the nose has a saddle-like appearance and the mandible exhibits prog- n al h ism. Potential problems in children with achondroplasia include overcrowding of the teeth, speech problems (articulatio ), and frequent ear infectior:Ls (otitis media). Dental malocclusion is treated \\. ith orthodontics. All children with achondroplasia should be evalu- ared b\ a speech therapist by two years of age because ofpossible problerrs with the de\eiopment ofclear speech. Articulation problems may be caused by onhodontic prob- Iems. Due to the abnormal shape of the eustachian tube in an individual with achondroplasia, they are very prone to ear infections (otitis media). \ote: The teeth are of nomal size but there is limited space within the maxillary and mandibular arches for them to erupt into which causes overcrowding and subsequent mal- o!'cluS ron.

What disorder is a result of a genetic mutation causing the substitution of glutamic acid by a valine and results in dental radiographs with enlarged marrow spaces? Cystic fibrosis Muscular dystrophy Polio Sickle-cell anemia

Sickle-cell anemia Sickle-cell anemia is an inherited disease in which the red blood cells, normally disc- shaped, become crescent shaped, As a result, they function abnomally and cause small blood clots. These clots give rise to recurrent painful episodes called crises." Sickle-cell anemia ofabnormal hemoglobin "sickle cell pain (also called sickle-cell disease) is the result ofthe production S/ due to a genetic defect. It is carried as a trait by 10% of African Americans and 0.2olo have sickle-cell anemia. lt is more common in fe- (Hemoglobin males and usually clinically manifests itselfbefore the age of30. The typical signs ofane- mia are present. The patient is weak, short ofbreath and easily fatigued. Muscle and patns are common. Denlal radiographs ar€ oft€n of diagnostic value: marrow spaces are markedly en- larged because ofthe loss ofmany trabeculae; the trabeculae, which are present, are often abnormally prominent. Occasionally, osteosclerotic areas are noted in the midst of large radiolucent marow spaces. However, the lamina dura and the teeth are unaffected. See picture #18 in booklet \ote: The gene defect is a known mutation ofa single nucleotlde (th),minelbr ofthe beta-globin gene, which results in glutamic acid to be substituted by valine. Sicklecell anemia occurs when a person inherits two abnormal ger'es (one front a person inherits one abnormal gene for the disease, they have what is called sickle cell trait. The life span ofred blood cells is reduced from 120 to 20 days.

Most cases of acute osteomyelitis are infections. Almost any organism may be part of the etiologic picture, although ? have been the most frequently cited. Bacteroides and campylobacter Clostridia and corynebacterium Staphylococci ard streptococci Enterococci and lactobacilli

Staphylococci ard streptococci Acutc inflammation ofthc bonc and bone manou'ofthe mandible and maxilla rcsults most fiequently from €rtension of a periapical abscess. The second most common cause of acute osteomyelitis is phlsical injury as secn u,ith fractlte or surgery. Most cases of acute osteomyelitis arc infcctious. Staphr-lococci and streptococci are the most frequcntly cited. Pain is the primary feature of this inflammatory process. Fever, painful lymphadenopathy, leukocyto- sis. and other signs and symptoms ofacutc infcction arc also commonly found. Paresthesia ofthe lotler lip is occasionally seen with mandibular involvcmcnt. Important: Unless the inflammatow process has bccn prcscnt for more than I wcck, radiographic cvidcnce ofacutc ostcomyelitis is usualll_ not present. \\'ith timc. diffuse radiolucent changes bcgin to appcar. Treatment includes antibiotics and drainage. Chronic osteomyelitis (chronic osteitis) fiay bc one ofthc scquclac ofacule osteomychlis /ei er ult- I eatetl o, in.tdequateb lrealed), or rt may represent a long-tenr, lorv-grade inflammatory reaction that nevcr $ ent rhrough a significant or clinically noticeable acute phase. Most investigators believe that bacleria /e.g., staph ococci, \treptococci, bacleroiles. acli on\rer'/ are responsible for thc vast major- iI ol chronic osteornyelitis cases. The mandible, especially the molar area, is much more lrequently jaw is a comrnonly encountered sign; loose te€th and sinus tracts are less frequently seen. Anesthesia is very uncommon. Radiographically, chronic osteomyclitis appears atl'ected rhan js thc rnaxilla. Pain is usually present, s\relling ofthe primarily as a radiolucent lesion that may show focal zones ofopacifica- rion. The luccnt pattem is often described as "moth-eaten" ance. Treatment includes antibiotics and scqucstrectomy. because ofits mottled radiographic appear- Gar16's osteomyelitis or chronic osteomyelitis with proliferative periosteitis, is cssentially a subtype of chronic osteomyclitis in which there is, additionally. a prominent periostcal inflammatory reaction. It periapical abscess of a mandibular molar in a child. The child characterisricali.v presents with an asymptomatic bony hard swelling with normal-appcaring overlying skin and mucosa. Radiographicallli thc lesion appears centrally as a mottled, predominantly lucent lesion in mosi olien results from a a pattem consistcnt uith chronic osteomyelitis. The featrre that provides thc distinctive difference is the periosteal reaction. This appears as an expanded cortex, oftcn uith concent c or parallcl opaque lay- els. Trabeculae perpendicular to the extraction and antibiotics. "onion skin" layers may also be present. Treatment includes tooth

The term used to describe a leukemia where leukemic cells appear in the blood but there is no significant lncrease in the number of white blood cells is called: . Aleukemic leukemia Subleukemic leukemia Stem cell leukemia

Subleukemic leukemia Other leukemia terms to know: . ".A.leukemic" leukemia is a term used to describe a lorm of leukemia in which there are leukemic cells present in the bone maffow, but the circulating white blood cells are n€ither immature nor increased in number. . Stem cell leukemia is a form of leukemia that is characterized by abnormal cells that are poorly differentiated but are considered to be precursors of lymphoblasts, myelo- blasts. or monoblasts. Note: these cells are too immature to classify. . A "leukemoid" qnd carbima- reaction is a term used to describe a marked increase in the number of circulating granulocytes. This condition is seen in a variety of disorders inch.rding cluonic infections and neoolasms.

A patient you saw yesterday had minor swelling of the submandibular space associated with a carious #31. You prescribed amoxicillin and sent him home. He called today to say the swelling has gotten worse. You squeeze him into the schedule and notice that he has trouble breathing. You call an ambulance to escort him to the ER and tell the paramedics that he has Ludwig's angina, Ludwig's angina is a severe and spreading infection that involves the: Submental and sublingual spaces only Submandibular, submental, and sublingual spaces unilaterally Submandibular and sublingual spaces only Submandibular, submental, and sublingual spaces bilaterally

Submandibular, submental, and sublingual spaces bilaterally Lud$ ig's angina oflen results fiom an odontogenic infection. As a result, the bacteriol- ogl of theses infections generally involves oral flora, particularly anaerobes. Other recounized etiologies of Ludwig's angina include poor oral hygiene, IV drug abuse, trauma, and tonsiliitis. It is characterized by: . . . . Rapid onset The three facial spaces are involved bilaterally Board-like swelling offloor olthe mouth and no fluctuance is present Tlpical "open mouthed" appearance . FIP\ siiAn of the tnnore . . . . Drooling. trismus, and fever Ditllculty eating, swallowing, breathing Tachvcardia. increased respiration rate Can lead to glottal edema; asphyxiation -\inr av management, massive antibiotic coyerage are the mainstays oftreatment. (IV), and surgical incision and drainage Verv important: The most serious complication of Ludwig's angina is edema of the glottis (i'hich is a slit-like opening behreen the true vocal corcls).

The most common type of malignant melanoma Superficial spreading Lentigo maligna Acral lentiginous Nodular

Superficial spreading Malignant melanoma involves the cells /melqnoc),tes) that produce pigment (zrelarirl, which is responsible for skin and hair color Melanoma can spread very rapidly and is the most deadly form ofskin cancer. lt is the leading cause of death from skin disease. Melanoma may appear on normal skin, or it may begin at a mole frevusl or other area that has changed in appearance. Some moles present at birth may develop into melanomas. The development of melanoma is related to sun exposure, particularly to sunbums during childhood, and is most common among people with fair skin, blue or green eyes, and red or blonde hair. There are four major types ofmelanoma; I . Superficial spreading melanoma is the most common type of melanoma. It is usually flat and irregular in shape and colol with varying shades ofblack and brown. lt may occur at any age or site and is most common in Caucasians. 2. Nodular melanoma usually starts as a raised area that is dark blackish-blue or bluish red, although some lack color Poorest prognosis, 3. Lentigo maligna melanoma usually occurs in the elderly. It is most cornmon in sun- damaged skin on the face, neck, and arms. The abnormal skin areas are usually large, flat, and tan with intermixed areas of brown. DeveJops from pre-existing lentigo maligna (Hutchinson freckle). Acral lentiginous melanoma is the least common form of melanoma. It usually occurs on rhe palms, soles, or under the nails and is more common in African Americans. ,1. Important: Malignant melanoma exhibits either initial radial (do not metastqsize) or c^l lnetastasis mdf occur) gtowthpattems within the skin. Radial growth phase melanomas include the superficial spreading, lentigo maligna, and acral lentiginous melanomas. Vertical qro\\th is characteristic of nodular melanoma. \ote: Skin cancer is a very common malignancy in the United States. Olthe different types, basal cell carcinoma is most common followed bv souamous cell carcinoma and the least common is malisnant melanoma.

A 6-year-old boy is a patient in your practice who has been hospitalized multiple times for broken bones. During routine lab tests, it was noted that his alkaline phosphatase levels were quite high. Dentally, you notice multiple impacted teeth. If this patient has a form offibrous dysplasia, which radiographic feature may you lind? The lesions are usually radiolucent, well-circumscribed, and may have a "cotton wool" appearance The lesions are usually a saucer-shaped radiolucency The lesions are usually radiopaque, not well-circumscribed, and may have a "ground- glass" appearance The lesions are usually well-demarcated unilocular or multilocular radiolucencies

The lesions are usually radiopaque, not well-circumscribed, and may have a "ground- glass" appearance Fibrous dysplasia is an idiopathic condition in which normal mcdullary bone is gradually replaced by an abnormal fibrous connective tissue proliferation. The mesenchymal tissue contains varying amounts of ostcoid and osseous material that prcsumably arises through mctaplasia. The discase most commonly presents as an asymptomatic, slow enlargement of thc involved bcrne. It may involve one bone or several boncs concomitantly. Forms of fibrous dysplasia: . \Ionostotic: is the most common, comprising 70o% ofcases. There is an equal male to fcmalc ratio, and this fclrm is the most likely to quiesce at puberty. A t!?ical monostotic lcsion will involve thc femur tibia. or ribs, uith 25% occuring in the bones ofthe skull. See picture #49 in booklet . Pol] ostotic fibrous dysplasia applies to cases in rvhich nrore than one bone exhibits evidcncc ofthc disorder. It is relatively uncommon; however, many patients have lesions of the skull, facial bones, t r.ia$ s. as a componcnt ofthe condition. Note: Alkaline phosphatase may bc elevated in up to 30% oipatients $ith polyostotic fibrous dysplasia, and a dramatic dse may herald malignant degeneration. Calcium and phosphorus tcnd to be normal. \otes- l. Craniofacial librous dysplasia is a form offibrous dysplasia charactcrized by maxillary lesions that extend to involve the maxillary sinus, zygoma, sphcnoid bone. and floor ofthe orbit. Itfccune-Albright syndrome is a designation that has been applied to patients with polyostotic fibrous dysplasia, cutaneous melanotic pigmentation endocrine abnormalities (most fcafe au lait macules), and commonlr precocious sexual development infenales). 3. Jaffe-Lichtenstein syndrome has been used to describe patients with multiple bone le- sions offibrous dysplasia and skin pigmentations. 4. Osteoblastoma is an uncommon primary lesion of bone that occasionally arises in the rnaxilla or mandible. It is a benign process that may exhibit a seemingly rapid onset and cause pain. The mandible is the most liequent head and neck sitc. Young adult malcs arc most commonly affected. Radiographically, is a well-circumscribed lesion that varies fiom radiolucent to radiopaque. May have a "sun-ray" paftcm ofnew bone production.

All of the following statements concerning metastatic tumors of the jaws are true EXCEPT They may be completely asymptomatic The patient is usually aware of slight discomfort or pain The maxilla is affected far more frequently than the mandible The molar region is predominantly involved

The maxilla is affected far more frequently than the mandible This is false; the mandible is affected far more ftequently than the n.raxilla. The most cornmon malignancy affecting skeletal bones is metastatic carcinoma. How- e\ er. netastatic disease to the mandible and maxilla is unusLra'l (onl.v about I %o). Most im- portantll, a tumor of the rurlrrr from its primary site. \ote: \letastases to the breasr jaws may be the first evidence of dissemination of a kno$'n jaws most commonly originate from primary carcinomas of the (ade o(arcinona), kidney (renal cell carcinona).lur'g tr()Ltr(i .) t.t). proslale (adenocarcinona) and thyroid gland creasing order of fiequency. Clinical features of metastatic . . Ofder adults jaw lesions: (average age is 561,ears) The mechanism ofspread to the ceral neoplasm . . \\'ithin the jar.v, jaws (carcinona), colon (ode ocarcinonn), in de- is usually hematogenous from the primary vis- the angle and body of the mandible are more commonly involved Bone pain, loosening of teeth, lip paresthesia, bone swelling, gingival mass, and pathologic fracture may be clinically evident . The radiographic appearance of mostjaw metastases is poorly marginated' radio- lucent, irregular, moth-eaten, expansile defects \ote: Microscopicalll', the diagnosis of metastatic carcinoma in difficult cases can be r erified u,ith an immunoperoxidase stain for cytok€ratin, which is present in all carci- noma cells. Important: The prognosis for patients with metastatic carcinoma to the is grave, rvith a disrnal l0olo 5-year survival and over two-thirds dead within a year.

When using the TNM method when assesing the diagnosis The presence of Nikolsky's sign The presence of nodules The presence of regional lymph node involvement The presence of erythroplakia

The presence of regional lymph node involvement Clinical staging of malignant neoplasms: The T:-Nl method of assessing the prognosis and thempy of malignant neoplasms is based upon 1) the size ofthe primary tumor, 2) the presence ofregional lymph node involvement and 3) the presence of distant metastases. This is represented as: . T . . : \ = \I size of the primary tumor presence ofregional lymph node involvement : presence ofdistant metastases \ote: The use of this classification system is becoming less common. A major short- corring ofthe TNM classification is the lack ofquantification of depth/penetration ofthe umor.

Which statement is true concerning chronic leukemias? They have a rapid onset and progression They have a shorter, more devastating clinical course than the acute leukemias They are characterized by proliferations more mature than those of the acute leukemias They constitute 75% of all leukemias

They are characterized by proliferations more mature than those of the acute leukemias Important: Chronic leukemias have a slower onset and progression. They also have a longer, less devastating clinical course than the acute leukemias, and they constitute about 507o of all leukemias. Other important clinical features of chronic leukemias: .Insidious onset witl't weakness and weight loss: disease may be detected during exanination for some other condition (e.g., anemia, unexplained hemorrhages, or rccw're nt intrqctab le inlbctio t1 . ) Organ involvement similar to acute type: massive splenomegaly is characteristic of chronic myelogenous leukemia; lymph node enlargement is main pathologic find- ing in lymphocytic tyPe . lymphocytic anemia may be complicated by autoimmune hemolytic anemia . Petechiae and ecchymoses' recurrent hemorhages, bactedal infections Laboratory findings: leukocylosis above 100,000 per cu. mm. with matur€ forms (gronulocltes ancl lymphocytes) predominating; Philad€lphia chromosome and low levels of leukocyte alkaline phosphatase are corunon in chronic myeloid l€ukemia (c.\'tL) . l \'ledian survival time for patients with chronic myelogenous leukemia ears rvith death <lue to hemorrhage or infection; chronic lymphocytic leukemia (C

Cancer of which oral surgery structure is most commonly associated with mortality Lip Tongue Floor of mouth Buccal mucosa

Tongue Cancer ofthe tongue causes more deaths than do malignant lesions in other regions ofthe head and neck. This has been attributed to the fact that it is a highly mobile organ that is richly en- dorved rvith lymphatics and blood vessels which facilitate metastases. IntraorallY. the most c'ommon sjte is the posterior lateral border of the tongue, it is panicularly uncommon to har e lesions develop on the dorsum or on the tip ofthe tongue. Remember: Metastases tiom ronsue cancer are relatively common at the time ofprimary treatment. In general. meiastatlc deposits from SCC ofthe tongue are found in the lymph nodes ofthe neck, usuaily on the ip- silateral side. The first nodes to become involved are the submandibular or nodes at the angle ofthe rnandible. Cancer ofthe lips: lip carcinomas account for 25 to 30% ofall oral cance$. Approximately 900 o occur on the lower lip due to increased sun exposure. Pipe smoking has also been im- phcated in the etiology. Cancer of the lloor of the mouth: is the second most common intraoral location of SCC. lt occurs most comlronly in the anterior segment on either side ofthe midline, near the orifices ofthe salivary glands. Metastasis to submandibular lymph nodes is not uncommon for lesions tbund here. Note: Prognosis is very poor for lesions found here. Cancer of the buccal mucosa: generally occurs along the plane of occlusion, midway an- teropostenorly. Cancer of the gingiva: is more common in the mandible than in the maxilla, and posterior siles are seen more frequently than anterior Note: Overall the 5-year suwival rate for SCC is 45% to 50%; with neck metastasis it is 25%.

Upon viewing a panorex ofa l4-year-old patient, you see a well-defined multilocular radiolucency with scalloping around the roots on the left side of the mandible apical to the canine and first premolar. No clinical symptoms are present. Teeth are not carious and respond normally to vitality tests. Medical history is unremarkable. Upon opening into the area, no fluid or tissue is evident. What is the most probable diagnosis? Dentigerous cyst Traumatic (simple) bone cyst Primordial cyst Residual cyst Stafne (static) bone defect

Traumatic (simple) bone cyst Remember: This cyst may be completely devoid ofsolid or liquid material. lt occurc most fre- quently in younger persons rlith no sex predilection. The usual location is in the mandible bc- tween the canine and ramus. The regional teeth are vital. Clinical features: . Children and adolescents. usually belbre the age of20 . . . . Usually asynlptomatic Prinarily seen in mandiblc Slight ilun) conieal bonc crpansion Associated with vital teeth, no displaced teeth The dentigerous cyst (or c.vst) contains a crown ofan unerupted tooth or denlal anonralv such as an odontoma. Enlarged dentigerous cysls can cause Drarked displaccmcnt oftccth. Prcs- lbllicular sure of accumulatcd fluid usually displaces the tooth in an apical direction. See picture #50 in booklet The primordial cyst diflers from the periodontal and dentigerous cysts in that it conlains no calcrlied structures. These cysts are lined by stratified squanous epithelium and may be either uniloc- ular. multilocular or multiple. The residual cyst rcl'ers to a situation in which a tooth associated with a radicular cyst is cxtractcd but the c)st is lcft undisturbed, it persists within thejaw and this lcsion is called a residual cyst. \ote: \bu must curette the socket ofa tooth with a radicular cyst aftcr extraction. See pictures # 86 and #87 in booklet Ihe st^Ine (stqtic) bone defect is radiolucency ofthe mandible due to invagination ofthc lingual .urlace of the of the jaw. lt is locatcd in thc posterior mandible below the mandibular canal. \ote: Thc focal osteoporotic bone marrow defect is a radiolucency in the jaw that contains henratopoietic bonc marrow. 11 is olten seen in an extraction site.

A 34-year-old dentophobe is your patient for the morning. After giving a very hesitant health history, you decide to begin your oral exam, As you reach toward her face, she immediately flinches and puts her hands up. She lets you know that ifyou tough a particular point above her lip, she gets sharp, stabbing jolts ofpain. You let her know that a neurologist can work her up for: Glossopharyngeal neuralgia Trigeminal neuralgia Postherpetic neuralgia Diabetic neuralgia

Trigeminal neuralgia Trigeminal neuralgia is an excruciating painful illness in which the afl-licted f'eels sudden stabJike pains in the face. The pains usually last only moments, but are among the most severe pains humans can feel. The pain is provoked by touching a n picallv near the nose or mouth. lt is caused by degeneration ofthe trigeminal nerve or b1 pressure being applied to it. Pain distribution is unilateral and follows the sensory dis- tribution of cranial nerve V, typically radiating to the maxillary / "trigger zone," (I'-2i or mandibular I --l/ area. At times, both distributions are affected. The momentary bursts ofpain recur in clusters. lasting many seconds. Paroxysmal episodes ofthe pains may last for hours. \eccssary criteria for diagnosis of trigeminal neuralgia: . . . . . Onser ofpain initiated by a trigger point Pain extreme. paroxysmal, lanclnatlng Durltron is less thrn 2 minutes -{ refractory period experienced for several minutes after attack Pain limited to krown distribution of one or more branches of trigeminal nerve with no motor deficit in that area . . Pain diminished with use ofcarbamazepine Spontaneous remissions occur lasting tnore than six months during the early phase of the disease The dnrg ofchoice for treating trigeminal neuralgia is carbzmazepine (Tegrelor. lt is an analgesic and anticonvulsant. It is also prescribed in the treatment ofcefiain seizure dis- orders. When used for the trcatment of trigeminal neuralgia, it usually rclieves the pain u'ithin 48 hours aftcr treatment is startcd.

A patient with which typ€ of amelogenesis imperfecta will have teeth that demonstrate enamel that varies from thin and smooth to normal thickness with grooves, furrows and/or pits? Type I (Hypoplastic) Type II (Hypomaturation) Type III (Hypocalcified)

Type I (Hypoplastic) (see chart for more details) Remember: L Amelogenesis imperfecta is a hereditary ectodermal defect, unlike dentinogenesis imper_ fecta which is a hereditary mesodermal f. In all three types ofame)ogenesis imperfecta, fected by the disease process itself defect. the dentin, pulp and cementum are unaf- /anlte dentinogenesis imperfecta).

Which type of dentinogenesis imperfecta features multiple pulp exposures, perirpical radiolucencies, and a variable radiographic appearance? Type I Type II Type III

Type III Dentinogenesis imp€rfecta is an autosomal dominant condition in which there is an intrinsic alteration in the dentin. lt affects dentin ofboth the primary and the permanent dentition. It is also known as hereditary opalescent dentin. Dentinogenesis imperfecta has been divided into three types: . Type I: dentin abnormality occurs in patients that have ost€ogenesis imperfecta fcra,'- In this form, primary teeth are more severely affected than permanent teeth. acterized blt blue sclera or a history of bone . frqchu?s). Tlpe lI: most common, only the dentin abnormality exists with no bone involvement. 'Tlpe III (Brandl'whe \pe) bke Type lI, only the dentin abnormality exists, however, there are clinical and radiographic vadations in this type. Features of type III that are not seen in type I and ll include multiple pulp exposures, periapical radiolucencies, and a variable radiographic appearance Important: Clinically, dentinogenesis imperfecta is usually easily detected and identified. The teeth exhibit an unusual translucent or opalescent appearance with color variation fiom yellorv-brorvn to gray. The entire crown appears discolored owing to the abnormal underlying dentin. See picture #3 in bookl€t Other characteristics ofthe teeth include: . Although the enamel is structurally and chemically normal, it fractures easily, resulting in rapid tvear . . . (due to poor dentin support) There is excessive constdction at the CEJ, giving the crowns a tulip or bell shape Short roots Obliterated pulps (\'pe I and I1 only) *** Note' In Type IlI, the dentin appean thin, and the pulp chambers and root canal are extremely large, giving the appearance ofthin dentin shells. See picture #4 in booklet Treatment is full crown coverage for cosmetic purposes. Note: These teeth should not be used as abutments, because the roots are prone to fractue under stress.

Scleroderma is a systemic disease that alfects many organ systems. The symptoms result from inflammation and progressive tissue fibrosis and occlusion ofthe microvasculature by excessive production and deposition of: Types II and IV collagens Types I and lll collagens Elastin and reticulin All of the above

Types I and lll collagens Otlrcr lcvels of nracromolecules found in the conncctivc tissue glytosantinoglycans, :a,1u'(tn. lihronecti ) are also incrcascd. fe.g., scleroderma is an uncommon. autoimmune disease. It all'ects the connective tissues $hich sur:rrund thcjoinls, blood vcsscls and intcmal organs bcneath the af}'ected area ofskin. Womcn arc af- i.tcd thrcc to ibur tinres morc oftcn than men. The diseasc usually starts between thc ages of25 llnJ :0 It onl-'- oqcasionally bcgins in children or in the clderly. Although there is no curc. propcr lrearnlent and care can makc it possible for people rvith sclerodenna to lead full, productive livcs. \ote: \1a) occur concomitantly with other autoimmune diseases. Such as lupus erythemalosus, rheunaroid arthritis. dermatomyositis, and Sjogren's sundrome. \ote: Thesc changcs in thc con- n.-iti\c trssue all'ect the fingers, trunk, face (producing u rr(]\]]'nal parts Reaclive lesion caused tmuma to oral mucous of the cxtrenlitieS. "purse-string" moil/i], and the nlore Oral radiographs ol'a patient with scleroderma would show an abnormal rvidening of the peri- ..Jrrnral ligament. This space is created by a thickening ofthe periodontal-membranc as a rcsult of :n increase in size and number ofcollagen fibers. The enlarged space is almost uniform io width, .rlrounds the cntirc root of the toolh and makes the tooth appear as if it is being cxlrudcd rapidly :ilrir its socket. See picture #31 in booklet Note: Other oral radiographic l'eatures may include ]rlar.ral resorption ofthe angle ofthe ramus ofthe mandible or complete resorption 01-thc condylcs :nd or coronoid process ofthe mandible. Remember: The abnormal widening ofthc periodontal:rembrane spacc is also a radiographic finding in osteosarcomas. Thcrc arc tlvo major types ofscleroderma: Painless, broad-bas€d swelling lhar is lighter rhan surrounding ii$ue; fi€quenlly fotmd on buccal mucosa, laleral border oftongxe, snd tower lip Orat l€iomyomas arc rare; present as slow-growlng, asymptomatic subrnucosal oasses, usualiy in the longue, bard palate, or buc€al mucosa I Localized scleroderma; In this condition changes only occur in isolatcd areas of the skin and the tissues beneath it. [t is rclativcly mild and docs not aft'cct intemal organs. Cutaneous changes includc induration and rigidity. atrophy, and telangiectasias. L Systemic scleroderma: With this condition changes rnay occur in thc skin and also in a num- ber of intemal organs. These might include blood vessels, joints, the digcstivc syst"em gus, stomach and borlel/, and occasionally the lungs, hean, kidneys and muscles. Changes in the connective tissue may affect the function ofany ofthese organs.

A 65-year-old patient ofEast Indian origin presents to your clinic with a complaint of a lesion on the maxillary alveolar tuberosity. The lesion is a thick white, exophytic mass with a cauliflower appearance. A social history reveals that this woman has been chewing a betel-nut concoction from her native India for over 40 years. What is the likely diagnosis of this lesion? Papilloma Erythroplakia Verrucous carcinoma Hyperkeratosis

Verrucous carcinoma -\ r'errucous carcinoma is a well-differentiated squamous cell neoplasm ofsoft tissue of the oral or laryngeal cavity. The lesion may invade or infiltrate the borders of adjacent structures but it rarely metastasizes. Verucous carcinoma may transform into an invasile fom of carcinoma or coexist with other squamous cell carcinomas. It is often misdiagnosed histologically as a benign lesion. Clinical Features: . . . . . . \{ale predilection Broad-based, exophytic, indurated lesion Diffuse, whitish, cauliflower or coral papillary mass \{andibular mucobuccal fold, alveolar mucosa and palate Slo*-growing, continuous enlargement Painless See picture #,13 in booklet \ote: Tobacco and human papillomavirus (subtypes 16 and 18)may be etiologic factors. It is treated by surgical excision and has a good prognosis.

A deficiency of parathyroid hormone can be treated with Vitarnin A Vitamin C Vitamin D Vitamin K

Vitamin D Hypoparathyroidism is a rare disorder associated with insufficient production ofparathyroid hormone, the inability to make a usable form ofparathy'roid hormone, or the inabil- ity ofkidneys and bones to respond to parathyroid hormone production. Hypoparathyroidism can result from congenital disorders, iatrogenic causes rcmoval ol the porathyroid glands during thyroid or parath!-roid sto'qery, radiation), rn- filtration ofthe parathyroid glands (e.g., (e.g., drugs, metqstatic cqrcinoma. Il/ilson disease. sarcoid), suppression of parathyroid function, HIV/AIDS, or idiopathic mechanisms. Hypocalcemia is the most important consequence of hypoparathyroidism. Symptoms occur when ionized calcium level drops to less than 2.5-3 mg/100 mL. The clinical man- ifestation is tetany. A positive Chvostek's sign rapped on the facial (nitching oJ the.fdcial muscles v'hen nen e near the parotid gland) is characteristic ofhypoparathyroidism. lmportant: The dental manifestations of hypoparathyroidism (i.e., delayed eruption, euantel ht,poplasia and blunted root apices) may be prevented by early treatment with vitamin D. Remember: The term muscular dystrophy refers to a group ofgenetic diseases marked by the progressive weakness and degeneration olthe skeletal, or voluntary muscles, which con- fiol molement. Oral manifestations include an increase in dental disease iforal hygiene is neglected. weakness in the muscles of mastication leading to decreased maxillary biting ibrce and a higher-incidence of mouth breathing and open bite.

On a hospital rotation you see an infant who displays bowed legs and muscle weakness and dentally you notice a delayed eruption pattern. The child has rickets which is a deficiency in what VitaminA Vitamin D Vitamin C Vitamin E

Vitamin D Osteomalacia involves softening of the bones caused by a deficiency of vitamin D or problems with the metabolism ofthis vitamin. This softening ofthe bones occurs because the bones contain osteoid tissue which has lailed to calcily due to the lack of Vitamin D. Note: Osteomalacia may occur as a complication ofsteatorrhea secondary to chronic pan- creatrtls. Clinical findings: . . . . Osteopenia Bone softening/deformity: hourglass thorax, bowing of long bones Increased fractures. biconcave vertebral bodies Mottled skull All bones are affected, specifically their epiphyseal growth plates. Osteomalacia appears to be more common in women. This condition mav be asvmDtomatic until fracture occurs. Blood tests may show: . Lou levels of vitamin D . . \la) also show low calcium and phosphorus levels .\lkaline phosphatase levels can be high Rickets is osteomalacia in children. It causes skeletal deformities. It is usually accompanied bl listlessness, initability and generalized muscular weakness. A child with rickets may hare borvlegs and develop a pigeon breast and a protruding stomach. The teeth in a child l ith rickets are affected as follows: delayed eruption, malocclusion and developmental abnormalities ofthe dentin and enamel alone with a hi{:her caries rate.

Leukemia is a group of bone mrrrow diseases involving an uncontrolled increase ln: Red blood cells (Erythocytes) Platelets Plasma cells White blood cells (Leukocytes)

White blood cells (Leukocytes) Leukemia is a lomr ofcancer that begins nbe") in appearance jn the blood-forming cells ofthe bone marrow soft, innet part o.l'1he botret. Undcr nomal circumstances, the blood-fonning, or hematopoictic, cclls of thc bone marrow make leukocytes to defend thc body against infectioLls organlsms such as r inrses and bacteria. But il'sot'ne leukocytes are damaged and remain in an inmlature fbrm' they becornc poor int'ection fighters that multiply excessivcly and do not dic olfas thcy should. The leukcmic cells accumulate and lesscn the production of oxygcn-carrying red blood cclls /ery- rlr r or r les/. blood-clo tting cells (plaletets), and normal lcukocytes. If untrcated, the surplus leukemic cells or er$helm the bone marrow, enter the bloodstream. and eventually invadc other parts ofthe btrdr. such as the lymph nodes, spleen, livcr, and central nervous syslern (brcin, "^Pfualcoldr' * ar. the bchar ior of leukcmia is different than that of other canccrs, which usually bcgin in major organs and ultimatcl), spread to thc bone marrow. Thcrc are more than a dozen varictics ofleukenia, but thc follorving four types arc thc most com- nlon: L -\cute lymphocytic lcnkcrlta (ALL). most common type in children 1. .\cute myelogenous le]rlemia j. -1. Chronic lymphocytic lcukernia (AML.): most malignant type fczlr: least malignant type Chronic myclogenous leukcmia (CML)t 2 distinct phases, invariably latal lmportant: . . . Leukemia is classified by thc dominant cell type and by the duration from onset to death Leukemia can modify thc inflammalory reaction Leukemia's affect on teeth/gingiva: Duc to a decrease in immune responsc, periodontal problenrs can be aggravated. Gingival enlargement can be a finding rvith leukemia Additionally, deficits with platelets can bc scen intraorally with increased hemorrhaging,/bruising."


संबंधित स्टडी सेट्स

Serve Safe 6th ed Ch 5 Study Questions

View Set

LAB 2: PowerLab 1: EEG, Psychophysiology and Wii Lab

View Set

Drivers ED Module 1 Topics 1 and 2

View Set

Chapter 1 - Introduction to Quantitative Analysis

View Set

Trennbare Verben, A1, alemán para hispanohablantes, verbos separables, Schritte International Neu 1, Lektion 5

View Set